Heme2 Exam 1 PQs

¡Supera tus tareas y exámenes ahora con Quizwiz!

A 28-year-old woman delivers a male neonate at 36 weeks of gestation. The mother has a history of poor prenatal care and several previous miscarriages. Examination of the neonate reveals marked pallor and generalized edema (anasarca), and the peripheral blood smear is shown in the image. The nucle- ated cells in this blood smear are which of the following? (A) Blymphocytes (B) Eosinophils (C) Erythroblasts (D) Monocytes (E) T lymphocytes

C: Erythroblasts. The peripheral blood smear displays erythroid precursors, which are normally confined to the bone marrow. Hemolytic disease of the newborn reflects a histoincompatibility between the mother and the developing fetus. The mother lacks an antigen that is expressed by the fetus. Maternal IgG alloantibodies cross the placenta, causing complement-mediated hemolysis of fetal erythrocytes and resulting in the release of numerous erythroid precursors (erythroblasts). The other choices represent normal immune cells. Diagnosis: Hemolytic disease of newborn, erythroblastosis fetalis

A 73-year-old woman has had increasing fatigue with a 3 kg weight loss over the past 7 months. Her hands become purple and painful upon exposure to cold. On physical examination she has a palpable spleen tip. Laboratory studies show Hgb 10.5 g/dL, Hct 31.7%, MCV 99 fL, platelet count 193,600/microliter, and WBC count 5390/microliter. The direct Coombs test is positive at 4°C and negative at 37°C. Which of the following underlying diseases is this woman most likely to have? A Non-Hodgkin lymphoma B Systemic lupus erythematosus C Pernicious anemia D Scleroderma E Thalassemia minor

*A. Non-Hodgkin lymphoma* She has a cold autoimmune hemolytic anemia. Though this may be idiopathic or due to an infection (Epstein-Barr virus; Mycoplasma) a malignant lymphoma must be considered. Since the RBC's are being sequestered in the spleen with extravascular hemolysis, there is splenomegaly. A warm autoimmune hemolytic anemia will often produce intravascular hemolysis with hemoglobinemia that will more often lead to renal failure. There will more often be jaundice with intravascular hemolysis. Intravascular hemolysis will decrease haptoglobin more. B - SLE is more likely to be associated with a warm immunohemolytic anemia. SLE is less common with advancing older age. C - Pernicious anemia should be associated with a higher MCV (this woman's slight increase in MCV could be explained by reticulocytosis). Megaloblastic anemias are not associated with immune hemolysis. D - Scleroderma can be associated with Raynaud phenomenon, but it is unlikely to be associated with an immunohemolytic anemia. E - Thalassemias can have ineffective erythropoiesis, but not immune-mediated hemolysis.

A 28-year-old African-American woman has had worsening fatigue for the past 2 months. On physical examination she has an erythematous macular rash on her upper chest, forearms, and face. Laboratory studies show Hgb 9.2 g/dL, Hct 27.9%, MCV 101 fL, platelet count 179,000/microliter, and WBC count 5850/microliter. The red blood cell distribution width is markedly increased. Her peripheral blood smear shows polychromasia. Her reticulocyte count is 4.2%. The serum haptoglobin is 3 mg/dL. Serum chemistries show total protein 7.9 g/dL, albumin 3.8 g/dL, alkaline phosphatase 49 U/L, AST 81 U/L, ALT 27 U/L, total bilirubin 3.3 mg/dL, and direct bilirubin 0.8 mg/dL. Hemoglobinuria is detected on urinalysis. Which of the following underlying conditions is she most likely to have? A Multiple myeloma B Systemic lupus erythematosus C Hepatitis C infection D Hereditary spherocytosis E Vitamin B12 deficiency

B Systemic lupus erythematosus These findings all point to intravascular hemolysis that can be seen with warm autoimmune hemolytic anemia that can complicate autoimmune diseases such as SLE. Her reticulocytosis is the response to the hemolysis and accounts for the mild increase in MCV. C - Chronic viral infections may lead to anemia of chronic disease, not to hemolysis. D - Abnormal RBC shapes typically lead to increased RBC destruction in the spleen (extravascular), not intravascular hemolysis. E - B12 deficiency leads to anemia, but not to hemolysis. In this case the slightly elevated MCV is from reticulocytosis.

A 10-year-old girl is noted to have increasing facial distortion for the past 8 months from a lesion involving her jaw. On physical examination she has a right mandibular mass. A biopsy is performed and on microscopic examination reveals a monotonous pattern of small non-cleaved lymphocytes. Cytogenetic analysis of these cells shows t(8;14). Infection with which of the following organisms is most likely to be associated with development of this girl's mass lesion? A Adenovirus B Cytomegalovirus C Epstein-Barr virus D Hepatitis C virus E Herpes simplex virus F HIV G HTLV-1

C Epstein-Barr virus Burkitt lymphoma is endemic in Africa, mainly seen in children and young adults, and EBV infection is implicated in the pathogenesis of this B-cell lymphoma. Although Burkitt lymphoma is one of the fastest growing neoplasms, half of cases do respond to chemotherapy with long-term survival.

A 10-year-old boy presents with chronic fatigue. Physical examination reveals slight jaundice and splenomegaly. The results of laboratory studies include hemoglobin of 11.7 g/dL, hematocrit of 32%, total bilirubin of 2.6 mg/dL, and conjugated bilirubin of 0.8 mg/dL. The peripheral blood smear is shown in the image. The osmotic fragility of the patient's RBCs is increased, but the Coombs test is negative. Defects in which of the following are involved in the pathogenesis of this disorder? (A) DNA synthesis (B) Erythrocyte cytoskeleton (C) Erythrocyte maturation (D) Glucose-6-phosphate dehydrogenase (G6PD) (E) Hemoglobin synthesis

C. Erythrocyte cytoskeleton. The smear shows many RBCs to be spherocytes, with decreased diameter and no central pallor. Hereditary spherocytosis (HS) represents a heterogeneous group of inherited disorders of the erythrocyte cytoskeleton, characterized by a defi ciency of spectrin or other cytoskeletal components (ankyrin, protein 4.2, band 3). Most forms of HS are inherited as autosomal dominant traits, and most patients have a moderate normocytic anemia. The bone marrow demonstrates erythroid hyperplasia (erythroid maturation is not affected). The deficiency of a cytoskeletal protein in HS leads to uncoupling of the lipid bilayer from the underlying cytoskeleton. The defect results in progressive loss of membrane surface area and formation of spherocytes, which show increased osmotic fragility and are susceptible to chronic extravascular hemolysis. The osmotic fragility test is not abnormal in G6PD defi ciency (choice D). Diagnosis: Hereditary spherocytosis

A 42-year-old man has had multiple episodes of painful red nodules on his skin from dermal venous thrombosis, as well as abdominal pain from mesenteric vein thrombosis over the past year. He notes passing darker urine. Laboratory studies show Hgb 9.4 g/dL, Hct 29.2%, MCV 100 fL, platelet count 215,000/microliter, and WBC count of 8800/microliter. His RBCs show increased sensitivity to complement lysis. Flow cytometry is most likely to show reduction in which of the following markers on his RBCs? A CD4 B CD19 C CD33 D CD55 E CD68

D CD55 He has paroxysmal nocturnal hemoglobinuria (PNH) an acquired stem cell disorder from mutation in the PIGA gene that renders RBCs very sensitive to complement lysis, as well as thrombosis in unusual veins. There is also risk for leukemia. The RBC markers CD55 and CD59 are reduced with PNH.

A 47-year-old man has been chronically fatigued for the past year. A physical examination yields no abnormal findings. A CBC shows: Hgb 10.8 g/dL, Hct 33.1%, MCV 104 fL, platelet count 239,000/microliter, and WBC count 7720/microliter. His peripheral blood smear shows normal WBC morphology and RBCs with mild poikilocytosis and a few target cells. His serum vitamin B12 is 512 pg/mL and folate 4.7 ng/mL. His serum haptoglobin is 151 mg/dL. Direct and indirect Coombs tests are negative. Which of the following underlying conditions is most likely to explain his findings? A Chronic lymphocytic leukemia B Peptic ulcer disease C Lead poisoning D Chronic alcohol abuse E Hereditary spherocytosis

D Chronic alcohol abuse There is mild macrocytosis with normal B12 and folate, without evidence for significant hemolysis. Liver disease can lead to RBC membrane abnormalities, with the presence of a few target cells on the peripheral blood smear as well as macrocytes. Alcohol can also directly affect RBC maturation. Chronic alcohol abusers may also have poor diets lacking in folate. An MCV >105 fL is much more likely to result from megaloblastic anemia (B12 or folate deficiency). A - CLL can be associated with hemolytic anemia, which is not present here. CLL is usually seen in older persons. B - Peptic ulcer disease should lead to blood loss with iron deficiency and a low MCV. C - Lead poisoning interferes with heme synthesis, leading to a hypochromic microcytic anemia. There may be mild hemolysis. RBCs can show basophilic stippling, a feature of toxic marrow damage. E - Spherocytosis leads to small RBCs with low MCV.

A 12-year-old girl has the sudden onset of severe abdominal pain and back pain. On physical examination her abdomen is diffusely tender, but there are no masses. She is afebrile. A CBC shows Hgb 6.5 g/dL, Hct 19.0%, MCV 99 fL, platelet count 149,000/microliter, and WBC count 11,200/microliter. Examination of her peripheral blood smear shows nucleated RBCs and sickled RBCs. Which of the following types of gene mutation is she most likely to have? A Deletion B Duplication C Insertion D Missense E Nonsense F Splice site G Tandem repeat

D Missense Missense mutations involve a change in a single base pair for a amino acid. The defect in sickle cell anemia is a single amino acid substitution (valine for glutamic acid), but the beta globin chain is still made. The gene mutation originated in places such as West Africa where selection pressure from falciparum malaria gave heterozygotes a selective advantage. The gene frequency may less commonly be present in some Mediterranean and eastern Arabian populations. There is enough fetal hemoglobin at birth and through infancy to prevent significant sickling with sickle cell disease.

A 20-year-old healthy man incurs blunt force trauma to the abdomen in a motor vehicle accident. On physical examination he has upper abdominal tenderness. An abdominal CT scan reveals a splenic hematoma. At laparotomy a splenectomy is performed. Following splenectomy, which of the following peripheral blood morphologic findings is most likely to be present? A Tear drop cells B Elliptocytes C Target cells D Macro-ovalocytes E RBC inclusions

E RBC inclusions Howell-Jolly bodies and Pappenheimer bodies are seen within RBC's in splenectomized patients. A functioning spleen would normally remove such inclusions. A - Tear drop shaped RBCs are most likely to appear with myelophthisic processes such as myelofibrosis. B - Elliptocytosis can be an obscure, incidental finding. C - Target cells can be seen with liver disease and some hemoglobinopathies such as hemoglobin C disease. D - Macro-ovalocytes are seen with megaloblastic anemias.

A 77-year-old woman notices that small, pinpoint-to- blotchy areas of superficial hemorrhage have appeared on her gums and on the skin of her arms and legs over the past 3 weeks. On physical examination, she is afebrile and has no organomegaly. Laboratory studies show a normal prothrom- bin time and partial thromboplastin time. CBC shows hemoglobin of 12.7 g/dL, hematocrit of 37.2%, MCV of 80 μm3, platelet count of 276,000/mm3, and WBC count of 5600/mm3. Platelet function studies and fibrinogen level are normal, and no fibrin split products are detectable. Which of the following conditions best explains these findings? A Chronic renal failure B Macronodular cirrhosis C Meningococcemia D Metastatic carcinoma E Vitamin C deficiency

E Vitamin C deficiency Platelet number and function in this case are normal, and there is no detectable abnormality in the extrinsic or intrinsic pathways of coagulation as measured by the prothrombin time or partial thromboplastin time. Petechiae and ecchymoses can result from increased vascular fragility, a consequence of nutritional deficiency (e.g., vitamin C), infection (e.g., meningococcemia), and vasculitic diseases. Chronic renal failure may depress platelet function. Chronic liver disease would affect the prothrombin time. Meningococcemia is an acute illness. Metastatic disease does not directly affect hemostasis, although extensive marrow metastases could diminish platelet production.

The pt described in the previous question is at risk for development of which of the following conditions? (A) Acute renal tubular necrosis (B) Cholelithiasis (C) Cirrhosis (D) Nephrolithiasis (E) Portal hypertension

(B) Cholelithiasis While circulating through the spleen, spherocytes lose additional surface membrane before they ultimately succumb to extravascular hemolysis and produce hyperbilirubinemia. Up to 50% of patients with spherocytosis develop cholelithiasis, with pigmented (bilirubin) gallstones due to the increased supply of bilirubin. Kidney stones (choice D) do not contain bilirubin. The liver (choices C and E) and kidney (choice A) are not affected by hereditary spherocytosis. Diagnosis: Hereditary spherocytosis

A 39-year-old woman presents with a 2-month history of upper abdominal pain, weakness, and fatigue. Physical exami- nation reveals marked pallor. Laboratory studies show micro- cytic, hypochromic anemia (hemoglobin = 8.5 g/dL) and mild thrombocytosis. Gastroscopy discloses a mucosal defect in the antrum measuring 1.5 cm in diameter. Which of the following best describes the pathogenesis of anemia in this patient? (A) Defective globin chain synthesis (B) Impaired heme synthesis (C) Poor utilization of iron stores (D) Synthesis of structurally abnormal hemoglobin molecules (E) Toxic damage to bone marrow stem cells

(B) Impaired heme synthesis The presence of a peptic ulcer incriminates gastrointestinal bleeding as the cause of anemia. The resulting iron deficiency interferes with heme synthesis and thus leads to impaired hemoglobin production and anemia. Defective globin chain synthesis (choice A) and synthesis of structurally abnormal hemoglobin molecules (choice D) are hemoglobinopathies. Poor utilization of iron stores (choice C) reflects sideroblastic anemia and anemia of chronic disease. Diagnosis: Iron deficiency anemia, peptic ulcer disease

A 51-year-old man has become increasingly fatigued for the past 10 months. On physical examination there are no abnormal findings. Laboratory studies show his Hgb is 9.2, Hct 27.9%, MCV 132 fL, platelet count 242,000/microliter, and WBC count 7590/microliter. Which of the following morphologic findings is most likely to be present on examination of his peripheral blood smear? A Hypersegmented neutrophils B Nucleated red blood cells C Blasts with Auer rods D Hypochromic, microcytic RBC's E Schistocytes

*A - Hypersegmented neutrophils* The increased MCV points to a macrocytic anemia such as a megaloblastic anemia that can also have hypersegmented PMN's from delayed maturation and reduced numbers of nuclear divisions. B - Although the MCV can be increased with a pronounced reticulocytosis, it is usually not that high. C - The WBC count is usually high when an acute leukemia with blasts is present. D - Microcytic RBC's have a low MCV. E - Schistocytes, or fragmented RBC's, have a wide size distribution (high RDW).

A 3-year-old boy from Sicily has a poor appetite and is underweight for his age and height. Physical examination shows hepatosplenomegaly. The hemoglobin concentration is 6 g/dL, and the peripheral blood smear shows severely hypochromic and microcytic RBCs. The total serum iron level is normal, and the reticulocyte count is 10%. A radiograph of the skull shows maxillofacial deformities and expanded marrow spaces. Which of the following is the most likely cause of this child's illness? A Imbalance in α-globin and β-globin chain production B Increased fragility of erythrocyte membranes C Reduced synthesis of hemoglobin F D Relative deficiency of vitamin B12 E Sequestration of iron in reticuloendothelial cells

*A. Imbalance in α-globin and β-globin chain production* This patient of Mediterranean descent has β-thalassemia major. In this condition, there is a severe reduction in the synthesis of β-globin chains without impairment of α-globin synthesis. The free, unpaired α-globin chains form aggregates that precipitate within normoblasts and cause them to undergo apoptosis. The death of RBC precursors in the bone marrow is called "ineffective erythropoiesis." Not only does this cause anemia, but it also increases the absorption of dietary iron, giving rise to iron overload, which results in hemochromatosis with infiltrative cardiomyopathy, hepatic cirrhosis, and "bronze diabetes" from pancreatic islet dysfunction. The severe anemia triggers erythropoietin synthesis, which expands the erythropoietic marrow. The marrow expansion encroaches on the bones, causing maxillofacial deformities. Extramedullary hematopoiesis causes hepatosplenomegaly. In comparison, the hemolytic anemia is mild in β-thalassemia minor, and there is very little ineffective erythropoiesis. Hemochromatosis is particularly detrimental to the liver and heart. Patients with chronic anemia may require RBC transfusions, which adds even more iron to body stores. The other listed options do not lead to a marked expansion of hematopoiesis.

A 30-year-old woman complains of recent easy fatigability, bruising, and recurrent throat infections. Physical examination reveals numerous petechiae over her body and mouth. Abnormal laboratory findings include hemoglobin of 6 g/dL, WBC of 1,500/mL, and platelets of 20,000/mL. The bone marrow is hypocellular and displays increased fat. What is the appropriate diagnosis? (A) Aplastic anemia (B) Iron-deficiency anemia (C) Megaloblastic anemia (D) Myelofibrosis with myeloid metaplasia (E) Pure red cell aplasia

*A: Aplastic anemia* Aplastic anemia is a disorder of pluripotential stem cells that leads to bone marrow failure. The disorder features hypocellular bone marrow and pancytopenia (decreased circulating levels of all formed elements in the blood). Most cases are idiopathic. The bone marrow in aplastic anemia shows variably reduced cellularity, depending on the clinical stage of the disease. There is a decrease in the number of cells of myeloid, erythroid, and megakaryocytic lineages, with a relative increase in lymphocytes and plasma cells. As the cellularity decreases, there is a corresponding increase in bone marrow fat. Anemia, leukopenia (primarily granulocytopenia), and thrombocytopenia characterize aplastic anemia. Patients with aplastic anemia present with weakness, fatigue, infection, and bleeding. Iron-defi ciency anemia (choice B) and megaloblastic anemia (choice C) are not characterized by a hypoplastic bone marrow. Myelofi brosis (choice D) shows increased connective tissue. Megakaryocytes and myeloid cells are not decreased in pure red cell aplasia (choice E). Diagnosis: Aplastic anemia

A 42-year-old woman has noticed during the past month that even minor bumps produce major bruises over her body. On physical examination she has areas of purpura on the skin of her arms and legs. She is afebrile. Laboratory studies shows her prothrombin time is 12.9 seconds (control 13 sec) and partial thromboplastin time 26.2 seconds (control 25 sec). Her CBC shows a Hgb of 11.1 g/dL, Hct 33.1%, MCV 84 fL, platelet count 790,000/uL, and WBC count 45,400/uL. A bone marrow biopsy is performed and on microscopic examination shows hypercellularity with myeloid and megakaryocytic hyperplasia. Which of the following is the most likely diagnosis? A Epstein-Barr virus infection B Myeloproliferative disorder C Drug reaction to recent antibiotic therapy D Wiskott-Aldrich syndrome E Megaloblastic anemia

*B - Myeloproliferative disorder* There is an increase in numbers of more than one cell line in the marrow. Paradoxically, there is bleeding because the platelets are functionally abnormal, even though there are lots of them. A - EBV infection is associated with infectious mononucleosis, a benign process, as well as some hematologic malignancies, but it is not typically associated with marrow hypercellularity. C - Drug reactions are typically toxicities associated with a decrease of one or more cell lines. D - This syndrome is accompanied by thrombocytopenia along with immunodeficiency. E - Though the marrow may become somewhat hypercellular with megaloblastic anemia, the cell counts in peripheral blood are not markedly increased.

A 44-year-old woman has a 2-week history of multiple ecchymoses on her extremities after only minor trauma. She also reports feeling extremely weak. Over the previous 24 hours, she has developed a severe cough productive of yellowish sputum. On physical examination, her temperature is 38.4° C, and she has diffuse crackles on all lung fields. Labora- tory studies show hemoglobin, 7.2 g/dL; hematocrit, 21.4%; MCV, 88 μm3; platelet count, 35,000/mm3; and WBC count, 1400/mm3 with 20% segmented neutrophils, 1% bands, 66% lymphocytes, and 13% monocytes. The reticulocyte count is 0.1%. Which of the following historical findings would be most useful in determining the cause of her condition? A Dietary habits B Exposure to medications C Family history of anemias D Menstrual history E Recent bacterial infection

*B. Exposure to medications* Her pancytopenia and absence of a reticulocytosis strongly suggest bone marrow failure. Aplastic anemia has no apparent cause in half of all cases. In other cases, drugs and toxins may be identified; drugs such as chemotherapeutic agents are best known for this effect. A preceding viral infection may be identified in some cases, but bacterial infections rarely cause aplastic anemias. Individuals with pancytopenia are subject to bleeding disorders because of the low platelet count and to infections because of the low WBC count. Dietary history would not be helpful because this patient's clinical and laboratory picture is not characteristic of iron deficiency or folate or vitamin B12 deficiency. The only known familial cause of aplastic anemia (Fanconi anemia) is rare. Menstrual history would be relevant if the patient had hypochromic microcytic anemia.

A clinical study is conducted involving adults from 18 to 80 years of age who underwent splenectomy for blunt force abdominal trauma. An age-matched control group of patients consists of patients who have congestive splenomegaly. The laboratory findings from these subjects are analyzed. Which of the following laboratory test findings is most likely to be observed only in the study group following splenectomy? A Thrombocytopenia B RBC Howell-Jolly bodies C Decreased RBC distribution width D Leukopenia E Nucleated RBCs

*B. RBC Howell-Jolly bodies* Increased numbers of red blood cell inclusions such as nuclear fragments (Howell-Jolly bodies) and degenerated hemoglobin (Heinz bodies) appear following splenectomy. The other listed foils require a spleen.

A 32-year-old woman from Hanoi, Vietnam, gives birth at 34 weeks' gestation to a markedly hydropic stillborn male infant. Autopsy findings include hepatosplenomegaly and cardiomegaly, serous effusions in all body cavities, and generalized hydrops. No congenital anomalies are noted. There is marked extramedullary hematopoiesis in visceral organs. Which of the following hemoglobins is most likely predominant on hemoglobin electrophoresis of the fetal RBCs? A Hemoglobin A1 B Hemoglobin A2 C Hemoglobin Bart's D Hemoglobin E E Hemoglobin F F Hemoglobin H

*C Hemoglobin Bart's* The infant had α-thalassemia major, which is most likely to occur in individuals of Southeast Asian ancestry, each of whose parents could have two abnormal α-globin genes on chromosome 16. A complete lack of α-globin chains precludes formation of hemoglobins A1, A2, and F. Only a tetramer of γ chains (Bart's hemoglobin) can be made, leading to severe fetal anemia. Inheritance of three abnormal α-globin chains leads to hemoglobin H disease, with tetramers of β chains; survival to adulthood is possible. Hemoglobin E disease produces mild hemolytic anemias

A 20-year-old thin fashion model complains that she cannot concentrate and is always tired. She has heavy menstrual bleeding every month but is otherwise healthy. The peripheral blood smear is shown in the image. Which of the following laboratory findings would be expected in this patient? (A) Hyperbilirubinemia (B) Increased serum ferritin (C) Low plasma iron saturation (D) Positive direct Coombs test (E) Vitamin B12 deficiency

*C: Low plasma iron saturation* The blood smear reveals microcytic, hypochromic erythrocytes, characteristic of iron deficiency anemia caused by inadequate uptake or, more often, excessive loss of iron. Women who have menorrhagia, especially those who consume restricted diets, are especially prone to iron deficiency anemia. Iron stores of the body are reduced, as evidenced by reduced levels of serum ferritin (not increased ferritin, choice B) and low iron saturation (iron/total iron binding capacity). None of the other laboratory findings would be expected in a patient with iron deficiency anemia. Diagnosis: Iron deficiency anemia

A 77-year-old man has experienced increasing malaise and a 6-kg weight loss over the past year. He has noted more severe and constant back pain for the past 3 months. On physical examination, his temperature is 38.7° C. His prostate is firm and irregular when palpated on digital rectal examination. There is no organomegaly. A stool sample is negative for oc- cult blood. Laboratory studies include a urine culture positive for Escherichia coli, serum glucose of 70 mg/dL, creatinine of 1.1 mg/dL, total bilirubin of 1 mg/dL, alkaline phosphatase of 293 U/L, calcium of 10.3 mg/dL, phosphorus of 2.6 mg/dL, and PSA of 25 ng/mL. CBC shows hemoglobin, 9.1 g/dL; hematocrit, 27.3%; MCV, 94 μm3; platelet count, 55,600/mm3; and WBC count, 3570/mm3 with 18% segmented neutrophils, 7% bands, 2% metamyelocytes, 1% myelocytes, 61% lymphocytes, 11% monocytes, and 3 nucleated RBCs per 100 WBCs. What is the most likely diagnosis? A Anemia of chronic disease B Aplastic anemia C Hemolytic anemia D Megaloblastic anemia E Myelophthisic anemia

*E - Myelophthisic anemia* His prostatic adenocarcinoma has metastasized to the bone marrow. High alkaline phosphatase, hypercalcemia, and a leukoerythroblastic pattern in the peripheral blood (immature WBCs and RBCs) are a consequence of the tumor acting as a space-occupying lesion. Such myelophthisic anemias also may be caused by infections. The anemia of chronic disease is mild. Aplastic anemias are unlikely to include leukoerythroblastosis. Hemolytic anemia should be accompanied by an increase in bilirubin and no abnormalities in calcium metabolism. The MCV in this case is not in the megaloblastic range.

For the past 4 months, a 62-year-old previously healthy man has noted increasing fatigue and shortness of breath with minimal exercise. He has felt some abdominal discomfort over the past month. On physical examination he has non-tender cervical lymphadenopathy. The liver span is 15 cm in the right mid-clavicular line; the edge is smooth and palpable just below right costal margin. The spleen is palpated 3 cm below left costal margin on inspiration. A CBC shows WBC count 23,100/microliter with 16 segs, 2 bands, 78 lymphs, and 4 monos, Hgb 11.9 g/dL, Hct 36%, MCV 90, and platelet count 277,300/microliter. The direct Coombs test is positive. Which of the following is the most likely diagnosis? A Leukemoid reaction B Chronic myelogenous leukemia C Acute myelogenous leukemia D Acute lymphocytic leukemia E Chronic lymphocytic leukemia F Systemic lupus erythematosus

*E. Chronic lymphocytic leukemia* Most of the circulating cells are small, mature lymphocytes with CLL. Most persons with CLL are older adults. CLL can have a tissue component called small lymphocytic lymphoma (SLL) with the same small lymphocytes infiltrating organs such as liver and spleen.

A 12-year-old boy has a history of episodes of severe abdominal, chest, and back pain since early childhood. On physical examination, he is afebrile, and there is no organomegaly. Laboratory studies show hemoglobin of 11.2 g/dL, platelet count of 194,000/mm3, and WBC count of 9020/mm3. The peripheral blood smear shows occasional sickled cells, nucleated RBCs, and Howell-Jolly bodies. Hemoglobin electrophoresis shows 1% hemoglobin A2, 6% hemoglobin F, and 93% hemoglobin S. Hydroxyurea therapy is found to be beneficial in this patient. An increase in which of the following is the most likely basis for its therapeutic efficacy? A Erythrocyte production B Overall globin chain synthesis C Oxygen affinity of hemoglobin D Production of hemoglobin A E Production of hemoglobin F

*E. Production of hemoglobin F* Children and adults with sickle cell anemia may benefit from hydroxyurea therapy, which can increase the concentration of hemoglobin F in RBCs, which interferes with the polymerization of hemoglobin S. However, the therapeutic response to hydroxyurea often precedes the increase in hemoglobin F levels. Hydroxyurea also has an anti- inflammatory effect, increases the mean RBC volume, and can be oxidized by heme groups to produce nitric oxide that promotes vasodilation. Because hemoglobin F levels remain high through the first 5 to 6 months of life, patients with sickle cell anemia typically do not manifest the disease during infancy. Because both β-globin chains are affected, no hemoglobin A1 is produced, and A2 levels are never high. Globin synthesis overall is not going to increase, and globin synthesis must be balanced to produce normal hemoglobin. The hemolysis associated with sickling promotes erythropoiesis, but the concentration of hemoglobin S is not changed. Hydroxyurea does not significantly shift the oxygen dissociation curve or change the oxygen affinity of the various hemoglobins.

A 10-month-old boy of Arabic extraction is brought to the physician by his parents who complain that their child is failing to thrive. Physical examination reveals splenomegaly and jaundice. A CBC shows a microcytic, hypochromic anemia (hemoglobin = 7.4 g/dL). Fetal hemoglobin accounts for most of the hemoglobin. A peripheral blood smear is shown in the image. Which of the following is the appropriate diagnosis? (A) G6PD deficiency (B) Hereditary elliptocytosis (C) Hereditary spherocytosis (D) Iron deficiency anemia (E) β-Thalassemia

*E: β-Thalassemia* The β-thalassemias are a heterogeneous group of disorders that most often arise secondary to point mutations affecting the β-globin gene. Accordingly, hemoglobin A (α2β2) is not formed. Unpaired α-chains precipitate in red blood cells, accounting for ineffective erythropoiesis and increased hemolysis. The blood smear shows features characteristic of thalassemia, including hypochromic and microcytic RBCs, with anisocytosis, poikilocytosis, and target cells. In homozygous β-thalassemia, fetal hemoglobin (hemoglobin F) accounts for most of the hemoglobin, although increased levels of hemoglobin A2 are also present. Symptoms of the disease appear early in life, and affected children require constant transfusions. A heterozygous state for thalassemia may provide a protective effect against malaria and increase the reproductive potential of heterozygotes, thereby explaining the persistence of thalassemic disorders. Hemoglobin F is not increased in choices A, B, or C. Diagnosis: Homozygous β-thalassemia

A 46-year-old man has had worsening arthritis and swelling of his feet for the past year. On physical examination he has rales audible in all lung fields. A chest radiograph shows cardiomegaly and pulmonary edema. Laboratory studies show Hgb 13.0 g/dL, Hct 39.1%, MCV 86 fL, platelet count 255,500/uL, and WBC count 5920/uL. His serum iron is 406 microgram/mL with iron binding capacity 440 microgram/mL and ferritin 830 ng/mL. Which of the following is the most likely diagnosis? A Beta-thalassemia B Autoimmune hemolytic anemia C Anemia of chronic disease D Polycythemia vera E Pernicious anemia F Hereditary hemochromatosis

*F - Hereditary hemochromatosis* Hereditary hemochromatosis results from increased iron absorbtion with markedly increased iron stores. The iron accumulation in tissues results in manifestations such as hepatomegaly, skin pigmentation, diabetes mellitus, heart disease, arthritis, and hypogonadism. A - The ineffective erythropoiesis leads to increased iron absorption with increased iron stores, but there is an accompanying anemia with beta-thalassemia. B - The hemolysis leads to increased iron recycling, since the red cells are not lost from the body, so iron stores are not significantly increased. C - Storage iron accumulates and is not effectively utilized with anemia of chronic disease. However, iron stores to not increase dramatically. An anemia is present. D - There is an increased RBC mass with primary polycythemia, but iron stores are not significantly increased. E - Ineffective megaloblastic erythropoiesis leads to increased iron absorption, but not dramatically, and a macrocytic anemia is present.

A 56-year-old woman suffers the sudden onset of head- ache and photophobia, and her condition worsens for the next 2 days. On physical examination, she has a temperature of 38° C and is disoriented. CBC shows hemoglobin of 11.2 g/dL, he- matocrit of 33.7%, MCV of 94 μm3, platelet count of 32,000/ mm3, and WBC count of 9900/mm3. The peripheral blood smear shows schistocytes. The serum urea nitrogen level is 38 mg/dL, and the creatinine level is 3.9 mg/dL. Which of the following is the most likely diagnosis? A Autoimmune hemolytic anemia B β-Thalassemia major C Disseminated intravascular coagulation D Idiopathic thrombocytopenic purpura E Paroxysmal nocturnal hemoglobinuria F Thrombotic thrombocytopenic purpura

*F - Thrombotic thrombocytopenia purpura* The diagnosis of thrombotic thrombocytopenic purpura (TTP) is based on finding a classic pentad: transient neurologic problems, fever, thrombocytopenia, microangiopathic hemolytic anemia, and acute renal failure. The diagnosis is suggested by decreased ADAMTS13 activity. There is a deficiency of ADAMTS13, which acts as a von Willebrand factor multimer protease. Most cases of TTP are idiopathic and associated with antibodies to ADAMTS13. Hereditary TTP may result from mutations in the ADAMTS13 gene. The abnormalities are produced by small platelet-fibrin thrombi in small vessels in multiple organs. The heart, brain, and kidney often are severely affected. Of the other choices, only disseminated intravascular coagulation is a microangiopathic hemolytic anemia, but the pentad of TTP is missing.

A 39-year-old man has experienced chronic fatigue and weight loss for the past 3 months. There are no remarkable findings on physical examination. Laboratory studies show hemoglobin, 10.0 g/dL; hematocrit, 30.3%; MCV, 91 μm3; platelet count, 240,000/mm3; WBC count, 7550/mm3; serum iron 80 μg/dL; total iron-binding capacity, 145 μg/dL; and serum ferritin, 565 ng/mL. Serum erythropoetin levels are low for the level of Hb and hepcidin levels are elevated. Which of the following is the most likely diagnosis? A Anemia of chronic disease B Aplastic anemia C Iron deficiency anemia D Megaloblastic anemia E Microangiopathic hemolytic anemia F Thalassemia minor

A Anemia of chronic disease The increased ferritin concentration and reduced total iron-binding capacity are typical of anemia of chronic disease, such as an autoimmune disease. Increased levels of cytokines such as interleukin-6 lead to increased hepatic production of hepcidin that stops ferroportin from releasing storage iron, promoting sequestration of storage iron, with poor use for erythropoiesis. Secretion of erythropoietin by the kidney is impaired. Various underlying diseases, including cancer, collagen vascular diseases, and chronic infections, can produce this pattern of anemia. Iron deficiency would produce a microcytic anemia, with a low serum ferritin level and reduced hepcidin production. Aplastic anemia is unlikely because the platelet count and WBC count are normal. Megaloblastic anemias are macrocytic without an increase in iron stores. Microangiopathic hemolytic anemias are caused by serious acute conditions such as disseminated intravascular coagulation; these patients have thrombocytopenia caused by widespread thrombosis. Thalassemia minor is uncommon and is not associated with a positive ANA test result.

A 33-year-old woman, G3, P0, who has had two spon- taneous abortions, is in the second trimester of her third pregnancy. An ultrasound at 18 weeks' gestation revealed symmetric growth retardation. She gives birth to a stillborn fetus at 25 weeks, and experiences sudden onset of dyspnea. A pulmonary ventilation/perfusion scan indicates a high probability of thromboembolism. Four months later, she experiences an altered state of consciousness and sudden loss of movement in the right arm. A cerebral angiogram shows occlusion of a branch of the left middle cerebral ar- tery. Laboratory findings show hemoglobin, 13.4 g/dL; he- matocrit, 40.3%; MCV, 91 μm3; platelet count, 124,000/mm3; WBC count, 5530/mm3; prothrombin time, 13 seconds; par- tial thromboplastin time, 46 seconds; positive anticardiolip- in antibody; positive serologic test result for syphilis; and negative ANA. Which of the following best explains these findings? A Antiphospholipid syndrome B Myeloproliferative disorder C Thrombophlebitis D Treponema pallidum infection E Trousseau syndrome F Von Willebrand disease

A Antiphospholipid syndrome Some patients with antiphospholipid syndrome (APS) have systemic lupus erythematosus, but others (such as this patient) do not. Arterial and deep venous thrombosis can occur in APS, with increased risk particularly of cerebral arterial thrombosis. Anticardiolipin antibody often leads to a false-positive serologic test result for syphilis (Treponema pallidum infection). Thrombocytopenia is often present. APS should be considered in women who have recurrent miscarriages. Polycythemia vera is a myeloproliferative disorder that predisposes to thrombosis, but this patient's hemoglobin value does not support this diagnosis. Thrombophlebitis occurs more frequently in pregnancy, but this explains only venous thrombosis and not the anticardiolipin antibodies. Trousseau syndrome, a hypercoagulable state associated with an underlying malignancy, can explain venous and arterial thromboses, but not the anticardiolipin antibodies. The patient's age argues against cancer. Von Willebrand disease is a bleeding disorder without thrombotic complications.

A 9-year-old boy has developed prominent bruises on his extremities over the past week. On physical examina- tion, he has ecchymoses and petechiae on his arms and legs. Laboratory studies show hemoglobin, 13.8 g/dL; hematocrit, 41.9%; MCV, 933μm3; platelet count, 11,300/mm3; and WBC count, 7720/mm . He had respiratory syncytial virus pneumo- nia 3 weeks ago. His condition improves with corticosteroid therapy. Which of the following abnormalities is most likely to cause his hemorrhagic diathesis? A Antiplatelet antibodies B Bone marrow aplasia C Glycoprotein IIb/IIIa dysfunction D Vitamin C deficiency E Von Willebrand factor metalloproteinase deficiency

A Antiplatelet antibodies Acute immune thrombocytopenic purpura (ITP) and chronic ITP are caused by antiplatelet autoantibodies, but the acute form is typically seen in children after a viral disease. If the bone marrow was aplastic, all cell lines should be reduced. Glycoprotein IIb/IIIa dysfunction/deficiency can be seen with Glanzmann thrombasthenia and chronic ITP. Scurvy caused by vitamin C deficiency leads to increased capillary fragility with ecchymoses, but not to thrombocytopenia. Von Willebrand factor metalloproteinase deficiency

A 38-year-old woman has become increasingly fatigued for the past 3 months. During the past week she has noted purple blotches on her skin. On physical examination there are purpuric areas of skin on her trunk and extremities. She has no hepatosplenomegaly and no lymphadenopathy. Laboratory studies show Hgb 6.8 g/dL, Hct 20.7%, MCV 91 fL, platelet count 28,760/microliter, and WBC count 1940/microliter. Which of the following is the most likely diagnosis? A Aplastic anemia B Myeloproliferative disorder C Immune thrombocytopenic purpura D Large B cell lymphoma E Hereditary spherocytosis

A Aplastic anemia There is evidence from the peripheral cytopenias for marked hypocellularity of the bone marrow with aplastic anemia. The spleen is of normal size with aplastic anemia. The 'aplasia' here refers to the three major cell lines: myeloid, and megakaryocytic as well as erythroid. B - The spleen in CML can reach massive proportions. C - The spleen is often normal in size with ITP, and there is thrombocytopenia, but not marked pancytopenia. D - Large cell lymphomas tend to be localized masses that do not fill up the bone marrow to cause pancytopenia. E - There is splenomegaly with HS with mild cytopenias. Removal of the spleen in adults with HS leads to a decrease in problems resulting from spherocytosis.

A 23-year-old woman in her 25th week of pregnancy has felt no fetal movement for the past 3 days. Three weeks later, she still has not given birth and suddenly develops dyspnea with cyanosis. On physical examination, her temperature is 37° C, pulse is 106/min, respirations are 23/min, and blood pres- sure is 80/40 mm Hg. She has large ecchymoses over the skin of her entire body. A stool sample is positive for occult blood. Laboratory studies show an elevated prothrombin time and partial thromboplastin time. The platelet count is decreased, plasma fibrinogen is markedly decreased, and fibrin split prod- ucts are detected. A blood culture is negative. Which of the fol- lowing is the most likely cause of her bleeding diathesis? A Consumption of coagulation factors B Defects in platelet aggregation C Increased vascular fragility D Reduced production of platelets E Toxic injury to the endothelium

A Consumption of coagulation factors The presence of thrombocytopenia, increased prothrombin and partial thromboplastin times, and fibrin split products, and the low fibrinogen concentration all suggest disseminated intravascular coagulation (DIC), which was most likely caused by a retained dead fetus. This obstetric complication can lead to DIC through release of thromboplastins from the fetus. The thromboplastins cause widespread microvascular thrombosis and consume clotting factors and platelets. There is no defect in platelet function. There is no damage to the vascular endothelium or vascular wall. Platelet production is normal, but platelets are consumed by widespread thrombosis of small vessels.

During the past 6 months, a 25-year-old woman has noticed a malar skin rash that is made worse by sun exposure. She also has had arthralgias and myalgias. On physical exami- nation, she is afebrile and has a pulse of 100/min, respirations of 20/min, and blood pressure of 100/60 mm Hg. There is ery- thema of skin over the bridge of the nose. No organomegaly is noted. Laboratory findings include positive serologic test re- sults for ANA and double-stranded DNA, hemoglobin of 8.1 g/dL, hematocrit of 24.4%, platelet count of 87,000/mm3, and WBC count of 3950/mm3. The peripheral blood smear shows nucleated RBCs. A dipstick urinalysis is positive for blood, but there are no WBCs, RBCs, or casts seen on microscopic exami- nation of the urine. Which of the following laboratory findings is most likely to be present? A Decreased haptoglobin B Decreased iron C Decreased reticulocytosis D Elevated D dimer E Elevated hemoglobin F F Elevated protoporphyrin

A Decreased haptoglobin Haptoglobin is a serum protein that binds to free hemoglobin. Ordinarily, circulating hemoglobin is contained within RBCs, but hemolysis can release free hemoglobin. The haptoglobin is used up as the amount of free hemoglobin increases. Systemic lupus erythematosus (SLE) is an autoimmune disease that can result in hemolysis by means of autoantibodies directed at RBCs, and the Coombs test result is often positive. SLE is best known to afflict young women, but it has a broad age range. Decreased iron can cause a hypochromic, microcytic anemia, but with hemolysis, the RBCs are recycled. Hemolysis is often accompanied by reticulocytosis if the marrow is intact and the iron is not lost. An elevated D-dimer level suggests a microangiopathic hemolytic anemia. Autoimmune diseases do not affect globin chain synthesis. Protoporphyrin can be increased with some forms of porphyria.

A 62-year-old man is taken to the emergency department in a state of inebriation. He is well known there because this scenario has been repeated many times over 15 years. On physical examination, he is afebrile. The spleen tip is palpable, and the liv- er edge is firm. Laboratory studies show hemoglobin of 8.2 g/dL, hematocrit of 25.1%, MCV of 107 μm3, platelet count of 135,000/ mm3, and WBC count of 3920/mm3. The peripheral blood smear shows prominent anisocytosis and macrocytosis. Polychromatophilic RBCs are difficult to find. A few of the neutrophils show six to seven nuclear lobes. Which of the following is the most likely explanation of these findings in his peripheral blood cells? A Diminished nuclear maturation from impaired DNA synthesis B Extravascular hemolysis of antibody-coated cells C Imbalance in synthesis of α-globin and β-globin chains D Increased susceptibility to lysis by complement E Reduced deformability of RBC membranes

A Diminished nuclear maturation from impaired DNA synthesis Chronic alcohol abuse can lead to folate deficiency, giving rise to megaloblastic anemia. Folic acid and vitamin B12 act as coenzymes in DNA synthetic pathways. A deficiency of either impairs the normal process of nuclear maturation. The hematopoietic cell nuclei remain large and primitive looking, giving rise to megaloblasts. The mature RBCs are larger than normal (macrocytes). Neutrophils often show defective segmentation, manifested by extra nuclear lobes. The nuclear maturation defect affects all rapidly dividing cells in the body. Patients with chronic alcohol abuse can have thrombocytopenia and leukopenia, often because of secondary hypersplenism (alcoholic cirrhosis, leading to splenomegaly). Polychromatophilic RBCs represent reticulocytes, and their number is reduced because of the failure of marrow to produce adequate numbers of RBCs despite anemia. Hemolytic anemias, in which antibody coats RBCs, can occur in autoimmune diseases, prior transfusion, and erythroblastosis fetalis. Hemoglobinopathies can produce a mild macrocytosis because more reticulocytes are released. An imbalance in α-globin and β-globin chain synthesis, seen in thalassemias, leads to microcytosis of RBCs. Complement lysis is enhanced in paroxysmal nocturnal hemoglobinuria, which results from mutations in the PIGA gene. Reduced RBC membrane deformability is seen in patients with abnormalities of cytoskeletal proteins, such as spectrin; the latter causes hereditary spherocytosis.

A 48-year-old woman has experienced increasing weak- ness and dyspnea for the past 5 months. On physical examination, her temperature is 37° C, pulse 100/minute, respiratory rate 19/min, and blood pressure 115/75 mm Hg. Auscultation of the lungs reveals bilateral basilar crackles. Muscle strength diminishes from 5/5 to 4/5 with repetitive movement of her arms. Her strength returns with administration of an acetyl- cholinestrase inhibitor. A chest CT scan reveals a 6-cm circumscribed anterior mediastinal mass. Which of the following findings is most likely to be present on microscopic examina- tion of her bone marrow biopsy? A Erythroid hypoplasia B Lymphocytosis C Megakaryocytic hyperplasia D Metastatic carcinoma E Myelofibrosis F Plasmacytosis

A Erythroid hypoplasia She has myasthenia gravis with thymoma and red cell aplasia. The edrophonium, an acetylcholinesterase inhibitor, will counteract the effect of the acetylcholine receptor antibodies of myasthenia gravis, but not improve muscle function with antibodies against voltage-gated calcium channels in Lambert-Eaton myasthenia syndrome (a paraneoplastic syndrome often associated with small cell lung carcinomas). The pulmonary findings suggest heart failure, and the tachycardia is consistent with high-output congestive heart failure from anemia. Pure red cell aplasia can be primary or arise secondarily to neoplasms, particularly thymic tumors, or autoimmune disorders. A lymphocytosis would be characteristic for lymphocytic leukemia, but this would not affect muscle strength, and lymphoblastic leukemia/lymphoma produces mediastinal masses in much younger persons. Megakaryocytic hyperplasia would be characteristic for peripheral consumption of platelets, with a disorder such as immune thrombocytopenic purpura (ITP), which is not likely to be associated with thymoma. Most thymomas act in a benign fashion and do not metastasize widely. Myelofibrosis could produce anemia with fatigue and weakness, but not predominantly with repetitive motion. Plasmacytosis is associated with myeloma that affects bone marrow, not the mediastinum.

A 28-year-old, previously healthy man has noted increasing fatigue for the past 6 months and formation of bruises after minimal trauma. Over the past 2 days, he has developed a cough. On physical examination, his temperature is 38.9° C, and he has diffuse rales in both lungs. He has no hepatosplenomegaly and no lymphadenopathy. Laboratory findings include a sputum culture positive for Streptococcus pneumoniae, hemoglobin of 7.2 g/dL, hematocrit of 21.7%, platelet count of 23,400/mm3, WBC count of 1310/mm3, prothrombin time of 13 seconds, partial thromboplastin time of 28 seconds, and total bilirubin of 1 mg/dL. The ANA test result is negative. What is the most likely explanation of these findings? A Hematopoietic stem cell defect B Hemolysis of antibody-coated cells C Increased susceptibility to lysis by complement D Metastatic adenocarcinoma to bone marrow E Secondary hypersplenism

A Hematopoietic stem cell defect Aplastic anemia leads to marked pancytopenia. Many cases are idiopathic, although some can follow toxic exposures to chemotherapy drugs or to chemicals, such as benzene. Some cases may follow viral hepatitis infections. An intrinsic defect in stem cells, or T lymphocyte suppression of stem cells, can play a role in the development of aplastic anemia. Hemolysis is unlikely because the bilirubin is normal, and there is no history of an autoimmune disease. An increased susceptibility to complement lysis occurs in paroxysmal nocturnal hemoglobinuria as a result of mutations in the PIGA gene. It is unlikely that the patient has metastatic disease at this age, with no prior illness; metastases are more likely to produce a leukoerythroblastic peripheral blood appearance. Sequestration of peripheral blood cells in an enlarged spleen could account for mild pancytopenia, but in this case the spleen is not enlarged.

An infant is born at 34 weeks' gestation to a 28-year-old woman, G3, P2. At birth, the infant is observed to be markedly hydropic and icteric. A cord blood sample is taken, and direct Coombs test result is positive for the infant's RBCs. Which of the following is the most likely mechanism for the findings in this infant? A Hemolysis of antibody-coated cells B Hematopoietic stem cell defect C Impaired globin synthesis D Mechanical fragmentation of RBCs E Oxidative injury to hemoglobin F Reduced deformability of RBC membranes

A Hemolysis of antibody-coated cells The infant most likely has erythroblastosis fetalis because maternal antibodies are coating fetal RBCs. A fetalmaternal hemorrhage in utero or at the time of delivery in a previous pregnancy (or with previous transfusion of incompatible blood) can sensitize the mother, resulting in production of irregular IgG antibodies. In subsequent pregnancies, these antibodies (in contrast to the naturally occurring IgM antibodies) can cross the placenta to attach to fetal cells, leading to hemolysis. In the past, most cases were caused by Rh incompatibility (e.g., Rh-negative mother, Rh-positive infant), but the use of RhoGAM administered at birth to Rhnegative mothers has eliminated almost all such cases when recognized. Other, less common blood group antigens can be involved in this process, however. The other conditions listed are not antibody mediated. A stem cell defect results in aplastic anemia and immunodeficiency. Impaired globin synthesis occurs in thalassemias. Mechanical fragmentation of RBCs is typical of microangiopathic hemolytic anemias, such as disseminated intravascular coagulation, which is more typical of pregnant women with obstetric complications. Oxidative injury to hemoglobin is typical of glucose- 6-phosphate dehydrogenase (G6PD) deficiency. Reduced RBC m embrane deformability is seen in patients with abnormalities in cytoskeletal proteins, such as spectrin; the latter causes hereditary spherocytosis.

A 60-year-old man presents with a 6-month history of increasing fatigue. Physical examination reveals marked pallor, and a CBC shows a macrocytic anemia. Which of the following is the most likely cause of anemia in this patient? (A) Alcoholism (B) Chronic disease (C) Iron deficiency (D) Renal disease (E) Thalassemia

A. Alcoholism. Macrocytic anemia may be caused by impaired DNA synthesis due to a deficiency of folic acid or vitamin B12. This results in abnormal nuclear development, which, in turn, leads to ineffective erythrocyte maturation and macrocytic anemia. Folic acid deficiency is most commonly due to inadequate dietary intake, which often develops in patients with poorly balanced diets (e.g., alcoholics). Other possible causes of macrocytic anemia include liver disease, hypothyroidism, and primary bone marrow disease. None of the other choices are associated with macrocytic anemia. Chronic disease (choice B) and renal disease (choice D) cause normochromic, normocytic anemia. Iron deficiency (choice C) and thalassemia (choice E) are microcytic anemias. Diagnosis: Macrocytic anemia

A 72-year-old Caucasian man has had increasing fatigue for the past year. On physical examination there are no abnormal findings. Laboratory studies show a Hgb of 9.1 gm/dL, Hct 27.9%, MCV 96 fL, WBC count 3700/microliter, and platelet count 125,000/microliter. The WBC differential count shows 53 segs, 5 bands, 2 metamyelocytes, 1 myelocyte, 32 lymphs, 7 monos, and 5 nucleated RBCs/100 WBCs. He has a negative direct and indirect Coombs test. Which of the following diseases is he most likely to have? A Metastatic carcinoma B Chronic alcoholism C Malabsorption D Hemoglobinopathy E Chronic blood loss F Systemic lupus erythematosus

A. Metastatic carcinoma Metastatic tumor involving marrow, or marrow fibrosis, is a 'myelophthisic' process that reduces normal hematopoiesis and leads to a peripheral 'leukoerythroblastic' picture with immature RBC's and WBC's in the peripheral blood, as seen here with nucleated RBCs and white cells even more immature than bands (metamyelocytes, myelocytes) on the smear. B - Macrocytosis is a common feature of alcoholism, generally from folate deficiency and liver disease, but not release of nucleated RBC's. C - Malabsorption could lead to iron or B12 or folate deficiencies, but the decreased marrow production of RBC's is not accompanied by release of nucleated RBC's. D - A hemoglobinopathy (such as sickle cell disease) severe enough to lead to release of nucleated RBC's would probably preclude survival to age 72. E - Chronic blood loss leads to iron deficiency, and this decreases marrow RBC production. Nucleated RBC's are not released in that circumstance, because there are not enough raw materials (iron) to support early release of RBCs. F - SLE and other autoimmune diseases may result in an autoimmune hemolytic anemia, with a positive Coombs test.

A 45-year-old man has had increasing abdominal discomfort with abdominal enlargement for the past two years. On physical examination, the spleen can be felt below the left costal margin. There is no fluid wave. An abdominal CT scan reveals massive (estimated 3000 gm size) splenomegaly. Laboratory data include Hgb 9 g/dL, WBC count 5,000/microliter, and platelet count 50,000/microliter. Which of the following underlying conditions is he most likely to have? A Myelofibrosis B Sickle cell anemia C Portal hypertension D Infectious mononucleosis E Hemochromatosis

A. Myelofibrosis Myeloproliferative disorders, and myelofibrosis in particular, are known to cause massive splenomegaly. As hematopoiesis is reduced in the marrow, it moves elsewhere (extramedullary hematopoiesis) such as the spleen. B - The spleen with sickle cell anemia becomes very small ('autosplenectomy'). C - A spleen will rarely be larger than 1000 gm in a patient (such as a chronic alcoholic with micronodular cirrhosis of liver) who has portal hypertension. D - The spleen may be slightly enlarged and tender, but is not massively enlarged with Epstein-Barr virus infection, which lasts a few weeks. E - There may be increased iron stores and the spleen may be some-what enlarged with hemochromatosis, but massive enlargement does not occur.

Which of the following best describes the pathogenesis of the hematologic disorder seen in the patient described in Question 23? (A) Clonal stem cell defect (B) C-myc translocation (C) Deletion of a portion of the β-globin gene (D) Functionalasplenia (E) Mutation of the T-cell receptor gene

A: Clonal stem cell defect. Myelodysplastic syndromes (MDS) are hematopoietic stem cell disorders that are characterized by a discrepancy between the paucity of peripheral blood elements and marked hyperplasia in the bone marrow. MDS may be either primary (de novo) or secondary (therapy related). Patients with secondary myelodysplasia usually have a history of chemotherapy, especially alkylating agents, or radiation therapy for the treatment of cancer. Other risk factors for MDS include viruses, benzene exposure, cigarette smoking, and Fanconi anemia. Diagnosis: Myelodysplasia

A 32-year-old man presents with mild fever and increasing fatigue. He is an immigrant from Russia and worked in a benzene factory. Physical examination does not reveal lymphade- nopathy or splenomegaly, but petechial skin lesions are noted. A CBC demonstrates severe pancytopenia, with normocytic red cell indices. A bone marrow biopsy is shown in the image. Which of the following is the most likely underlying mechanism in the development of this patient's anemia? (A) Damage to stem cells (B) Decreased erythropoietin production by the kidneys (C) Folatedeficiency (D) Impaired globin chain synthesis (E) Neoplastic proliferation of committed stem cells

A: Damage to stem cells. The bone marrow is aplastic, consisting largely of fat cells and lacking normal hematopoietic activity. Patients with aplastic anemia present with severe pancytopenia and clinical symptoms related to the various cytopenias, including fatigue (anemia), fever (neutropenia), and petechiae (thrombocytopenia). The lack of an appropriate reticulocyte response to the anemia indicates decreased or ineffective hematopoiesis as the mechanism for the pancytopenia. Injury to bone marrow stem cells is idiopathic (two thirds of cases), toxic (as in this case), immunologic, or hereditary (Fanconi anemia). Diagnosis: Aplastic anemia

A 23-year-old, previously healthy man of Italian origin develops moderate to severe hemolytic anemia. The previ- ous evening he had celebrated a Saint's day with a feast of beans and pasta. Urinalysis shows free hemoglobin, and the direct Coombs test is negative. Supravital staining of the blood smear demonstrates numerous membrane-bound inclusions (Heinz bodies) within erythrocytes. Which of the following is the most likely diagnosis? (A) G6PD deficiency (B) Paroxysmal nocturnal hemoglobinuria (C) Sickle cell anemia (D) β-Thalassemia minor (E) Warm antibody autoimmune hemolytic anemia

A: G6PD deficiency. G6PD defi ciency is an X-linked disorder that causes a hemolytic anemia characterized by abnormal sensitivity of red cells to oxidative stress. The highest prevalence is in Africa and the Mediterranean region. Because of the role of G6PD in recycling reduced glutathione, red cells defi cient in this enzyme are susceptible to oxidative stress, which, in this case, is fava bean ingestion (favism). In quiescent periods, the erythrocytes of G6PD deficiency appear normal. However, during a hemolytic episode precipitated by oxidative stress, Heinz bodies can be demonstrated by supravital staining. Full expression of G6PD defi ciency is seen only in males, with females being asymptomatic carriers. Heinz bodies are not characteristic of the other choices. Diagnosis: Glucose-6-phosphate dehydrogenase deficiency

A 19-year-old primigravida of Southeast Asian ancestry gives birth at 35 weeks gestation a male infant. On physical examination the infant is markedly hydropic. Laboratory studies show his hematocrit is 17% and the peripheral blood smear reveals numerous nucleated red blood cells and even a few erythroblasts. The red blood cells display marked anisocytosis and poikilocytosis. Which of the following diseases is most likely to be present in this infant? A Sickle cell anemia B Alpha-thalassemia C Hemoglobin E disease D G6PD deficiency E Hereditary elliptocytosis

B Alpha-thalassemia The form of alpha-thalassemia seen in Southeast Asia has two abnormal alpha globin genes on the same chromosome, so it is possible to inherit two bad sets, leading to a complete lack of alpha globin chain formation, so that fetal hemoglobin (two alpha and two gamma chains) cannot be made, and there is a severe anemia. A - Sickle cell anemia is seen in persons of African and eastern Arabian ancestry. In perinatal life and in infancy, there is sufficient hemoglobin F to prevent complications. C - Hemoglobin E disease is seen in Southeast Asia, but typically causes a mild hemolytic anemia of adults. D - G6PD disease is most frequent in Africa and Mediterranean regions and can produce a hemolytic anemia when the RBC's are stressed by oxidants. E - Hereditary elliptocytosis may cause a mild anemia.

A 16-year-old boy has had a low energy level for as long as he can remember. On physical examination he has a palpable spleen tip. A CBC shows Hgb of 8.8 g/dL, Hct 24.1%, MCV 65 fL, platelet count 187,000/microliter, and WBC count 7400/microliter. His serum ferritin is 3740 ng/mL. A bone marrow biopsy is performed and on microscopic examination reveals a myeloid:erythroid ratio of 1:4, and there is 4+ stainable iron. Which of the following is the most likely diagnosis? A G6PD deficiency B Beta-thalassemia C Sickle cell anemia D Hereditary spherocytosis E Malaria

B Beta-thalassemia Reduced beta-globin chain synthesis from beta-thalassemia leads to RBC microcytosis, hypochromia, ineffective erythropoiesis, and excessive iron absorption. There is chronic anemia, because the major hemoglobin A1 is produced insufficiently. The nature of the mutation, typically affecting RNA transcript production, determines the severity of the disease. A - Glucose-6-phosphate dehydrogenase deficiency is episodic and usually not manifested until an oxidizing drug such as primaquine is ingested that depletes RBC glutathione activity. C - The spleen is typically very small by age 16 with sickle cell disease as a consequence of multiple vaso-occlusive crises with infarction. D - Hereditary spherocytosis is usually a mild disease, and there is not excessive iron absorbtion. E - There can be hemolysis and splenomegaly with malaria, but excessive iron absorption is not usually a finding. Such a degree of severity of malaria would be uncommon.

An 83-year-old man complains of worsening malaise and fatigue over the past 5 months. On physical examination, he is afebrile and normotensive. The spleen tip is palpable. A CBC shows hemoglobin, 10.6 g/dL; hematocrit, 29.8%; MCV, 92 μm3; platelet count, 95,000/mm3; and WBC count, 4900/mm3 with 63% segmented neutrophils, 7% bands, 2% metamyelocytes, 1% myelocytes, 22% lymphocytes, 5% mono- cytes, and 3 nucleated RBCs per 100 WBCs. The peripheral blood smear shows occasional teardrop cells. An examination of the bone marrow biopsy specimen and smear is most likely to show which of the following findings? A Erythroid hyperplasia B Extensive fibrosis C Fatty replacement D Many megaloblasts E Numerous myeloblasts

B Extensive fibrosis Teardrop RBCs are indicative of a myelophthisic disorder (i.e., something filling the bone marrow, such as fibrous connective tissue). The leukoerythroblastosis, including immature RBCs and WBCs, is most indicative of myelofibrosis. Splenomegaly also is typically seen in myelofibrosis. A leukoerythroblastic picture also can be seen in patients with infections and metastases involving the marrow. Hyperplasia of erythroid normoblasts occurs in hemolytic anemias. Leukoerythroblastosis is not seen in hemolytic anemias. Replacement of marrow by fat occurs in aplastic anemia, which is characterized by pancytopenia. The presence of megaloblasts in the marrow indicates folate or vitamin B12 deficiency—both cause macrocytic anemia. Marrow packed with myeloblasts is typical of acute myeloid leukemia. In this condition, the peripheral blood also would show many myeloblasts and failure of myeloid maturation.

A 17-year-old adolescent has had malaise for the past 3 weeks. He has a mild pharyngitis on physical examination, as well as tender axillary and inguinal lymphadenopathy. The spleen is palpable. A CBC shows Hgb 14.0 g/dL, Hct 42.2%, MCV 90 fL, platelet count 301,300/microliter, and WBC count 8120/microliter with 'atypical lymphocytes' on the peripheral blood smear. His illness is most likely to be acquired via which of the following mechanisms? A Congenital genetic abnormality B From close contact on a date C As a result of an insect bite D Through an environmental exposure at work E Without any known etiology

B From close contact on a date Infectious mononucleosis with Epstein-Barr virus (EBV) infection is typically acquired with close personal contact. This self-limited viral infection resolves in weeks.

A 68-year-old man has had malaise for the past year. On physical examination, there are no abnormal findings. His Hgb is 10.5 g/dL, Hct 31.5%, MCV 88 fL, platelet count 211,000/microliter, and WBC count 6980/microliter. His total serum iron is 130 microgm/dL total iron binding capacity (TIBC) 230 microgm/dL, and soluble serum transferrin receptor is normal. A bone marrow biopsy is performed and microscopic examination shows that maturation is occurring in all cell lines and there are no abnormal cells seen. Stainable iron in the bone marrow is increased. Which of the following underlying diseases is he most likely to have? A Diverticulosis B Hepatitis C infection C Systemic lupus erythematosus D Atrophic gastritis E Fanconi anemia

B Hepatitis C infection He has findings of anemia of chronic disease, with a high (or normal) % iron saturation, increased iron stores, and normal soluble serum transferrin receptor. In iron deficiency, iron stores are diminished, % saturation decreased, and soluble serum transferrin receptor increased. Hepatitis C viral infection is a chronic disease. A - Diverticular disease may lead to chronic blood loss with iron deficiency anemia, in which the TIBC should be increased and the RBCs should be microcytic. C - SLE can lead to cytopenias and hemolytic anemia and is more common in younger persons. He has no findings such as skin rash, arthralgia, or myalgia. Of the autoimmune diseases, rheumatoid arthritis is most likely to be associated with anemia of chronic disease. D - Atrophic gastritis should produce vitamin B12 deficiency with a megaloblastic anemia. E - This is a rare congenital anemia starting in childhood, with underlying defects in DNA repair, with associated anomalies such as organ hypoplasias.

A 29-year-old rugby player takes part in a particularly con- tentious game between New Zealand and South Africa. He is the forward prop in the scrums, hitting hard and being hit hard by other players. He feels better after downing several pints of beer following the game, but notes darker urine. Urinalysis is posi- tive for blood. Which of the following pathogenic mechanisms underlies change in the color of urine? A Complement lysis B Intravascular disruption C Osmotic fragility D Sinusoidal sickling E Splenic sequestration

B Intravascular disruption Mechanical trauma to RBCs is possible, but typically is not severe. It can follow strenuous exercises involving repeated blows to body parts. Complement-mediated lysis is a feature of immunohemolytic anemias. Increased osmotic fragility is noted in spherocytes. Sickle cell anemia is not likely to be found in the population groups in the countries noted, and persons with this disease are not likely to be playing rugby. Splenic sequestration is a feature of hemolytic anemias due to membrane defects and antibodies.

A 29-year-old woman has the sudden onset of fever, abdominal pain, tachycardia, and nausea. On physical examination her vital signs include T 37.6°C, P 90/minute, RR 18/minute, and BP 100/60 mm Hg. Her conjunctivae are icteric. The spleen tip is palpable. Laboratory studies shows Hgb 9.0 g/dL, Hct 27.3%, MCV 99 fL, platelet count 209,500/microliter, and WBC count 6840/microliter. Her reticulocyte count is 0.1%. On microscopic examination of her peripheral blood smear, the RBC's are small and lack central pallor. Which of the following most likely initiated this woman's acute illness? A Quinacrine use B Parvovirus infection C Decreased oxygen tension D Exposure to cold E Transfusion therapy

B Parvovirus infection The findings point to hereditary spherocytosis. Parvovirus B19 infects erythroid precursors and can lead to an aplastic crisis in persons with hemoglobinopathies. The hemoglobinopathy impairs the marrow ability to respond to the stress of the acute infection. A - This oxidant drug can lead to increased RBC spherocytosis with RBC destruction in the spleen because glutathione metabolism is impaired in persons with G6PD deficiency. However, there should be a reticulocytosis in response to the hemolysis. C - Decreased oxygen tension is the mechanism behind the shape change of RBC's with sickle cell anemia. Persons with sickle cell anemia may also have an aplastic crisis with parvovirus infection. D - Exposure to cold can lead to hemolyis in patients with cold agglutinins, so called cold autoimmune hemolytic anemia. E - Transfusion of normal RBC's will tend to improve patients with RBC defects. A hemolytic crisis from mismatched blood is an extreme rarity.

A 22-year-old woman has experienced malaise and a sore throat for 2 weeks. Her fingers turn white on exposure to cold. On physical examination, she has a temperature of 37.8° C, and the pharynx is erythematous. Laboratory find- ings include a positive monospot (heterophile antibody) test result. Direct and indirect Coombs test results are positive at 4° C, although not at 37° C. Which of the following molecules bound on the surfaces of the RBCs most likely accounts for these findings? A α2-macroglobulin B complement C3b C fibronectin D histamine E IgE

B complement C3b Cold agglutinin disease has antibody (usually IgM) coating RBCs. The IgM antibodies bind to the RBCs at low temperature at peripheral body sites and fix complement; however, complement is not lytic at this temperature. With an increase in temperature within core internal organs, the IgM is dissociated from the cell, leaving behind C3b. Most of the hemolysis occurs extravascularly in the cells of the mononuclear phagocyte system, such as Kupffer cells in the liver, or splenic macrophages, because the coating of complement C3b acts as an opsonin. IgG is typically involved in warm antibody hemolytic anemia, which is chronic and is not triggered by cold. Raynaud phenomenon occurs in exposed, colder areas of the body, such as the fingers and toes. The patient probably has an elevated cold agglutinin titer. Histamine is released in type I hypersensitivity reactions. Fibronectin is an adhesive cell surface glycoprotein that aids in tissue healing. IgE is present in allergic conditions.

A 17-year-old girl has had a history of fatigue and weak- ness for her entire life. She has not undergone puberty. On physical examination, secondary sex characteristics are not well developed. She has hepatosplenomegaly. CBC shows he- moglobin of 9.1 g/dL, hematocrit of 26.7%, MCV of 66 μm3, platelet count of 89,000/mm3, and WBC count of 3670/mm3. The appearance of the peripheral blood smear is shown in the figure. Additional laboratory findings include serum glucose of 144 mg/dL, TSH of 6.2 mU/mL, and ferritin of 679 ng/mL. A mutation in a gene encoding for which of the following is most likely to be present in this girl? A Ankyrin B β-Globin C G6PD D HFE E NADPH oxidase

B β-Globin This patient has β-thalassemia, probably of at least intermediate severity. There is decreased β-globin chain formation, with increased hemoglobin A2 and F to compensate. There is ineffective erythropoiesis and increased erythropoietin to drive increased iron absorption, leading to iron overload. Chronic anemia requiring transfusion therapy exacerbates hemochromatosis. Iron deposited in endocrine tissues can lead to gonadal, pituitary, thyroid, islet cell, and adrenal failure. Secondary hypersplenism can result from the splenomegaly, with sequestration of platelets and leukocytes. The abnormal ankyrin gene leads to hereditary spherocytosis and a mild hemolytic anemia with splenomegaly, but not to iron overload. In glucose-6-phosphate dehydrogenase deficiency, sensitivity to oxidizing agents causes a hemolytic anemia, but this usually is not ongoing. The HFE gene is abnormal in hereditary hemochromatosis, leading to iron overload, but onset of the disease occurs in middle age. Mutations involving NADPH oxidase lead to immunodeficiency in chronic granulomatous disease.

A 27-year-old pregnant woman comes to the obstetrician for a prenatal check-up. Routine laboratory testing reveals a mild normocytic anemia. The peripheral blood smear is shown in the image. Which of the following best explains the pathogenesis of anemia seen in this patient? (A) Abnormal membrane lipoprotein molecules (B) Abnormal polymerization of spectrin molecules (C) Decreased iron release in the bone marrow (D) Destabilization of the lipid bilayer of the RBC membrane (E) Oxidative denaturation of hemoglobin

B: Abnormal polymerization of spectrin molecules. The smear displays elliptical erythrocytes. Hereditary elliptocytosis (HE) refers to a heterogeneous group of inherited disorders involving the erythrocyte cytoskeleton, all of which feature a horizontal abnormality within the cytoskeleton. Variants of HE include defects in self-assembly of spectrin, spectrin-ankyrin binding, protein 4.1, and glycophorin C. Diagnosis: Hereditary elliptocytosis

The patient described in Question 16 is at increased risk of developing which of the following conditions? (A) Cerebral aneurysm (B) Cerebrovascular accident (C) Cholelithiasis (D) Osteogenic sarcoma (E) Raynaud phenomenon

B: Cerebrovascular accident. The patient has polycythemia vera (PV). Hyperviscosity associated with PV increases the risk for thrombotic stroke. The other choices are not associated with PV. Diagnosis: Polycythemia vera

A 45-year-old chronic alcoholic man presents with mental confusion. The peripheral blood smear is shown in the image. The morphologic abnormalities demonstrated in this blood smear are most likely associated with which of the following conditions? (A) Abnormal spectrin in red cell membranes (B) Chronic liver disease (C) Chronic renal failure (D) Microthrombi in capillaries (E) Vitamin B12 deficiency

B: Chronic liver disease. Acanthocytosis (shown in the photomicrograph) results from a defect within the lipid bilayer of the red cell membrane and features spiny projections of the surface, which may be associated with hemolysis. The most common cause of acanthocytosis is chronic liver disease, in which increased free cholesterol is deposited within the cell membrane. Abnormalities in the lipid membrane cause erythrocytes to become deformed and develop irregular spiny surface projections and centrally dense cytoplasm (acanthocytes or spur cells). Chronic renal failure (choice C) features burr cells. Abnormal spectrin (choice A) causes hereditary spherocytosis. Diagnosis: Acanthocytosis

A patient with a history of chronic alcoholism presents with a macrocytic anemia and thrombocytopenia. Blood smear examination demonstrates numerous oval macrocytes and hypersegmented neutrophils (results shown in the image). A Schilling test is normal. Which of the following is the most likely diagnosis? (A) Anemia of chronic disease (B) Folic acid deficiency (C) G6PDdeficiency (D) Irondeficiencyanemia (E) Sickle cell anemia

B: Folic acid defi ciency. Folic acid defi ciency commonly occurs in alcoholics who have poor nutrition. Macrocytosis, hypersegmented neutrophils, and a normal Schilling test (vitamin B12 absorption) point to folic acid deficiency. Folic acid and vitamin B12 are required for synthesis of DNA, and defi ciency of either factor leads to megaloblastic transformation of hematopoietic cells. Macrocytosis and hypersegmented neutrophils are not features of the other choices. Diagnosis: Megaloblastic anemia

A 23-year-old African-American man passes dark red- dish brown urine 3 days after taking an anti-inflammatory medication that includes phenacetin. He is surprised, because he has been healthy all his life and has had no major illnesses. On physical examination, he is afebrile, and there are no re- markable findings. CBC shows a mild normocytic anemia, but the peripheral blood smear shows precipitates of denatured globin (Heinz bodies) with supravital staining and scattered "bite cells" in the population of RBCs. Which of the following is the most likely diagnosis? A α-Thalassemia minor B β-Thalassemia minor C Glucose-6-phosphate dehydrogenase deficiency D Sickle cell trait E Abnormal ankyrin in RBC cytoskeletal membrane F Warm antibody autoimmune hemolytic anemia

C Glucose-6-phosphate dehydrogenase deficiency Glucose-6-phosphate dehydrogenase (G6PD) deficiency is an X-linked disorder that affects about 10% of African- American males. The lack of this enzyme subjects hemoglobin to damage by oxidants, including drugs such as primaquine, sulfonamides, nitrofurantoin, phenacetin, and aspirin (in large doses). Infection can also cause oxidative damage to hemoglobin. Heinz bodies are denatured hemoglobin, and they damage the RBC membrane, giving rise to intravascular hemolysis. The "bite cells" result from the attempts of overeager splenic macrophages to pluck out the Heinz bodies, adding an element of extravascular hemolysis. Heterozygotes with α-thalassemia (1 or 2 abnormal genes out of 4 total α-globin genes) have no major problems, but in cases of α-thalassemia major, perinatal death is the rule. Likewise, β-thalassemia minor and sickle cell trait are conditions usually with no major problems and no relation to drug usage. RBC membrane abnormalities, such as hereditary spherocytosis (caused by abnormal spectrin), typically produce a mild anemia without significant hemolysis, and there is no drug sensitivity. Some autoimmune hemolytic anemias can be drug related, but the hemolysis is predominantly extravascular.

A 72-year-old man has been feeling tired for the past 8 months. On physical examination there are no abnormal findings. Laboratory studies show Hgb 10.4 g/dL, Hct 30.3%, MCV 72 fL, platelet count 239,000/uL, and WBC count 7500/uL with automated differential count of 70.1% grans, 18.8% lymphs, and 11.1% monos. His total bilirubin is 1.0 mg/dL. Which of the following morphologic findings is most likely to be seen on his peripheral blood smear? A Fragmentation B Many nucleated forms C Hypochromasia D Spherocytosis E Howell-Jolly bodies

C - The most probable cause of a hypochromic, microcytic anemia in an older man is iron deficiency anemia. A - Fragmented RBC's, called schistocytes, are indicative of a microangiopathic hemolytic anemia such as DIC. B - Nucleated RBC's suggest hemolysis (which is not consistent with his bilirubin level) or a disease infiltrating and replacing bone marrow. D - Hereditary spherocytosis is uncommon. With a normal bilirubin, hemolysis from increased red blood cell turnover is probably not present. E - RBC inclusions suggest absence of the spleen.

A 65-year-old man has experienced worsening fatigue for the past 5 months. On physical examination, he is afebrile and has a pulse of 91/min, respirations of 18/min, and blood pressure of 105/60 mm Hg. There is no organomegaly. A stool sample is positive for occult blood. Laboratory findings include hemoglobin of 5.9 g/dL, hematocrit of 18.3%, MCV of 99 μm3, platelet count of 250,000/mm3, and WBC count of 7800/mm3. The reticulocyte concentration is 3.9%. No fibrin split products are detected, and direct and indirect Coombs test results are negative. A bone marrow biopsy specimen shows marked erythroid hyperplasia. Which of the following conditions best explains these findings? A Aplastic anemia B Autoimmune hemolytic anemia C Chronic blood loss D Iron deficiency anemia E Metastatic carcinoma

C Chronic blood loss The marked reticulocytosis and marrow hyperplasia indicate that the marrow is responding to a decrease in RBCs. The reticulocytes are larger RBCs that slightly increase the MCV. An aplastic marrow is very hypocellular and unable to respond to anemia; it is associated with pancytopenia. The normal Coombs test results exclude an autoimmune hemolytic anemia. Iron deficiency impairs the ability of the marrow to mount a significant and sustained reticulocytosis. Iron deficiency anemia is typically microcytic and hypochromic, but could be partially masked here by reticulocytosis, which would not be as marked if iron were not available, but his diet is supplying needed iron. Infiltrative disorders, such as metastases in the marrow, would impair the ability to mount a reticulocytosis of this degree.

A clinical study of patients who inherit mutations that reduce the level of ankyrin, the principal binding site for spec- trin, in the RBC membrane cytoskeleton shows an increased prevalence of chronic anemia with splenomegaly. For many patients, it is observed that splenectomy reduces the severity of anemia. This beneficial effect of splenectomy is most likely related to which of the following processes? A Decrease in opsonization of RBCs and lysis in spleen B Decrease in production of reactive oxygen species by splenic macrophages C Decrease in splenic RBC sequestration and lysis D Increase in deformability of RBCs within splenic sinusoids E Increase in splenic storage of iron

C Decrease in splenic RBC sequestration and lysis In patients with hereditary spherocytosis, spheroidal cells are trapped and destroyed in the spleen because the abnormal RBCs have reduced deformability. Splenectomy is beneficial because the spherocytes are no longer detained by the spleen. Splenectomy has no effect on the synthesis of spectrin or RBC deformability; the RBCs in spherocytosis are not killed by opsonization. In warm antibody hemolytic anemias, opsonized RBCs are removed by the spleen. Reactive oxygen species do not play a role in anemias. Iron is not the rate-limiting step to RBC production when the iron can be recycled within the body.

A clinical study of patients who inherit mutations that reduce the level of ankyrin, the principal binding site for spec- trin, in the RBC membrane cytoskeleton shows an increased prevalence of chronic anemia with splenomegaly. For many patients, it is observed that splenectomy reduces the severity of anemia. This beneficial effect of splenectomy is most likely related to which of the following processes? A Decrease in opsonization of RBCs and lysis in spleen B Decrease in production of reactive oxygen species by splenic macrophages C Decrease in splenic RBC sequestration and lysis D Increase in deformability of RBCs within splenic sinusoids E Increase in splenic storage of iron

C Decrease in splenic RBC sequestration and lysis In patients with hereditary spherocytosis, spheroidal cells are trapped and destroyed in the spleen because the abnormal RBCs have reduced deformability. Splenectomy is beneficial because the spherocytes are no longer detained by the spleen. Splenectomy has no effect on the synthesis of spectrin or RBC deformability; the RBCs in spherocytosis arenot killed by opsonization. In warm antibody hemolytic anemias, opsonized RBCs are removed by the spleen. Reactive oxygen species do not play a role in anemias. Iron is not the rate-limiting step to RBC production when the iron can be recycled within the body.

A 37-year-old woman has noted an excessively heavy menstrual flow each of the past 6 months. She also has noticed increasing numbers of pinpoint hemorrhages on her lower ex- tremities in the past month. Physical examination shows no organomegaly or lymphadenopathy. CBC shows hemoglobin of 14.2 g/dL, hematocrit of 42.5%, MCV of 91 μm3, platelet count of 15,000/mm3, and WBC count of 6950/mm3. On admission to the hospital, she has melena and after a transfusion of plate- lets, her platelet count does not increase. Which of the follow- ing describes the most likely basis for her bleeding tendency? A Abnormal production of platelets by megakaryocytes B Defective platelet-endothelial interactions C Destruction of antibody-coated platelets by the spleen D Excessive loss of platelets in menstrual blood E Suppression of pluripotent stem cell division

C Destruction of antibody-coated platelets by the spleen This patient's bleeding tendency is caused by a low platelet count. She most likely has idiopathic chronic immune thrombocytopenic purpura (ITP), in which platelets are destroyed in the spleen after being coated with antibodies to platelet membrane glycoproteins IIb-IIIa or Ib-IX. These antibodies coat both the patient's platelets and any transfused platelets. Because the spleen is the major source of the antibody and the site of platelet destruction, splenectomy can be beneficial if corticosteroid therapy is not. There is no defect in the production of platelets. Platelet functions are normal in ITP. Chronic blood loss would not lead to thrombocytopenia when normal bone marrow function is present. Abnormal platelet-endothelial interactions are more likely to cause thrombosis. Suppression of pluripotent stem cells gives rise to aplastic anemia, which is accompanied by pancytopenia.

A 13-year-old boy has the sudden onset of severe ab- dominal pain and cramping accompanied by chest pain, non- productive cough, and fever. On physical examination, his temperature is 39° C, pulse is 110/min, respirations are 22/min, and blood pressure is 80/50 mm Hg. He has diffuse abdominal tenderness, but no masses or organomegaly. Laboratory studies show a hematocrit of 18%. The peripheral blood smear is shown in the figure. A chest radiograph shows bilateral pulmonary in- filtrates. Which of the following is the most likely mechanism for initiation of his pulmonary problems? A Chronic hypoxia of the pulmonary parenchyma B Defects in the alternative pathway of complement activation C Extensive RBC adhesion to endothelium D Formation of autoantibodies to alveolar basement membrane E Intravascular antibody-induced hemolysis

C Extensive RBC adhesion to endothelium The crescent-shaped RBCs (sickled RBCs) are characteristic of hemoglobin SS. This disease is most common in individuals of African and eastern Arabian descent. The sickled RBCs are susceptible to hemolysis (mainly vascular, in the spleen), but they also can cause microvascular occlusions anywhere in the body, most commonly bone, lungs, liver, and brain, leading to ischemia and severe pain. Vascular occlusions in the lungs are often accompanied by infection and lead to "acute chest syndrome." Abdominal pain and back pain are common and severe, requiring prompt and effective analgesia. The cell membranes of reversibly sickled cells are abnormally "sticky," and they adhere to capillary endothelium, especially in lungs. Vasoconstriction is caused by depletion of NO by free hemoglobin. Adhesion of RBCs to endothelium retards blood flow, creates hypoxia, and precipitates local sickling and vascular occlusion. Chronic tissue hypoxia does occur in sickle cell anemia, but it produces insidious impairment of function in organs such as heart, kidneys, and lungs. Defects in the alternative pathway of complement activation predispose to infection with encapsulated bacteria, such as Haemophilus influenza and Streptococcus pneumoniae. Autoantibodies to alveolar basement membrane can be part of Goodpasture syndrome, which also affects kidneys. The most severe intravascular hemolysis occurs with major transfusion reactions.

A clinical study is performed using adult patients diagnosed with peptic ulcer disease, chronic blood loss, and hypochromic microcytic anemia. Their serum ferritin levels average 5 to 7 ng/mL. The rate of duodenal iron absorption in this study group is found to be much higher than in a nor- mal control group. After treatment with omeprazole and clar- ithromycin, study group patients have hematocrits of 40% to 42%, MCV of 82 to 85 μm3, and serum ferritin of 30 to 35 ng/ mL. Measured rates of iron absorption in the study group after therapy are now decreased to the range of the normal controls. Which of the following substances derived from liver is most likely to have been decreased in the study group patients be- fore therapy, and returned to normal after therapy? A Divalent metal transporter-1 (DMT-1) B Hemosiderin C Hepcidin D HLA-like transmembrane protein E Transferrin

C Hepcidin Iron absorption from the gut is tightly controlled. When body iron stores are adequate, absorption of dietary iron via DMT-1 in the duodenum is retarded, and release of iron from storage pools is inhibited. When body iron stores decrease, as with chronic blood loss, iron absorption increases. The liver-derived plasma peptide hepcidin has been found to be the iron absorption regulator. Hepcidin levels increase when iron stores are high. Such fine control of iron absorption may fail, as in patients with ineffective erythropoiesis (e.g., β-thalassemia) who continue to absorb iron despite excess storage iron. Hepcidin levels are inappropriately low with both hereditary and acquired hemochromatosis. DMT-1 is an iron transporter that moves nonheme iron from the gut lumen to duodenal epithelium. Hemosiderin is an aggregated form of ferritin that does not circulate and is found only in tissues. Mutations in the HFE gene, which encodes an HLA-like transmembrane protein, lead to excessive absorption of dietary iron and hemochromatosis. Transferrin transports iron between plasma, iron stores, and developing erythroblasts.

A 33-year-old previously healthy man with persistent fever and heart murmur is diagnosed with infective endocar- ditis. He receives a high dosage of a cephalosporin antibiotic during the next 10 days. He now has increasing fatigue. On physical examination he has tachycardia and scleral icterus. Laboratory studies show a hemoglobin level of 7.5 g/dL, platelet count of 261,000/mm3, and total WBC count of 8300/ mm3. The direct Coombs test is positive. The periperal blood smear shows reticulocytosis. Which of the following is the most likely cause for his anemia? A Dietary nutrient deficiency B Disseminated intravascular coagulopathy C Immune-mediated hemolysis D Infection with parvovirus E Inherited hemoglobinopathy F RBC cytoskeletal protein disorder

C Immune-mediated hemolysis Drug-induced hemolytic anemias are neither common nor severe enough to be recognized, since the hemolysis is mainly extravascular. However, many patients receive drugs, so the potential for a drug reaction exists, and this immune-mediated mechanism must be distinguished from other causes for anemia. Cephalosporins are the most frequent drugs implicated. Treatment consists of cessation of therapy with the drug, because most cases are due to drug dependent antibody formation. Nutrient deficiencies reduce marrow production, so a reticulocytosis is unlikely. DIC is unlikely with a normal platelet count. Parvovirus infection may suppress erythropoiesis transiently in individuals with normal red cells, but may precipitate an aplastic crisis in those with a hemoglobinopathy. Persons with abnormal red cells are likely to have a history of anemia. Hemoglobinopathies are not Coombs positive.

A 45-year-old woman has experienced worsening arthritis of her hands and feet for the past 15 years. On physical examina- tion, there are marked deformities of the hands and feet, with ulnar deviation of the hands and swan-neck deformities of the fin- gers. Laboratory studies show an elevated level of rheumatoid factor. CBC shows hemoglobin, 11.6 g/dL; hematocrit, 34.8%; MCV, 87 μm3; platelet count, 268,000/mm3; and WBC count, 6800/mm3. There is a normal serum haptoglobin level, serum iron concentration of 20 μg/dL, total iron-binding capacity of 195 μg/dL, percent saturation of 10.2, and serum ferritin concentration of 317 ng/mL. No fibrin split products are detected. The reticulocyte concentration is 1.1%. What is the most likely mechanism underlying this patient's hematologic abnormalities? A Autoantibodies against RBC membranes B Impaired synthesis of β-globin chains C Inadequate usage of stored iron D Mutation in the phosphatidylinositol glycan A (PIGA) gene E Sequestration of RBCs in splenic sinusoids F Space-occupying lesions in the bone marrow

C Inadequate usage of stored iron The iron concentration and iron-binding capacity are low; however, in contrast to the finding in anemia of iron deficiency, the serum ferritin level is increased. This increase is typical of anemia of chronic disease. In this case, the chronic disease is rheumatoid arthritis. Underlying chronic inflammatory or neoplastic diseases increase the secretion of cytokines such as interleukin-1, tumor necrosis factor, and interferon-γ. These cytokines promote sequestration of iron in storage compartments and depress erythropoietin production. Autoantibody hemolytic anemias occur in several autoimmune diseases, such as systemic lupus erythematous, but not usually in patients with rheumatoid arthritis, as in this case. Normal serum haptoglobin rules out intravascular hemolysis; iron is recycled at a rapid rate. Impaired synthesis of β-globin chains gives rise to β-thalassemias, also characterized by hemolysis. Complement lysis is enhanced in paroxysmal nocturnal hemoglobinuria, which results from mutations in the PIGA gene. Patients with this disorder have a history of infections. Sequestration of RBCs in the spleen occur when RBC membranes are abnormal, as in hereditary spherocytosis or sickle cell anemia, or RBCs are coated by antibodies, as in autoimmune hemolytic anemias. Metastases are space-occupying lesions (myelophthisic process) that can lead to leukoerythroblastosis, with nucleated RBCs and immature WBCs appearing on the peripheral blood smear.

A clinical study is performed to assess outcomes in pa- tients who have macrocytic anemias as a result of Vitamin 12 or folate deficiency. A comparison of laboratory testing strategies shows that the best strategy includes testing for serum homocysteine, methylmalonic acid, vitamin B12 (cobalamin), and folate. What is the most important reason for ordering these tests simultaneously? A Aplastic anemia can result from lack of either nutrient B Both nutrients are absorbed similarly C Neurologic injury must be prevented D Life-threatening thrombocytopenia can occur in both E Therapy for one deficiency also treats the other

C Neurologic injury must be prevented Although folate and vitamin B12 deficiency both give rise to a macrocytic anemia, a deficiency of vitamin B12 also can result in demyelination of the posterior and lateral columns of the spinal cord. In some cases this deficiency will only be revealed by elevated levels of homocysteine and methylmalonic acid in the serum, because these are more sensitive indicators, particularly earlier in the disease. The anemia caused by vitamin B12 deficiency can be ameliorated by increased administration of folate; this masks the potential neurologic injury by improving the anemia. Treating vitamin B12 deficiency does not improve the anemia caused by folate deficiency, however. An aplastic anemia is unlikely to result from a nutritional deficiency. Folate has no cofactor for absorption, but vitamin B12 must be complexed to intrinsic factor and secreted by gastric parietal cells, and then the complex must be absorbed in the terminal ileum, so diseases such as atrophic gastritis and Crohn disease can affect vitamin B12 absorption more than folate. The peripheral smear could appear the same and offers no means for distinguishing these deficiencies.

A 54-year-old, previously healthy man has experienced minor fatigue on exertion for the past 9 months. On physical examination, there are no remarkable findings. Laboratory studies show hemoglobin of 11.7 g/dL, hematocrit of 34.8%, MCV of 73 μm3, platelet count of 315,000/mm3, and WBC count of 8035/mm3. Which of the following is the most sensi- tive and cost-effective test that the physician should order to help to determine the cause of these findings? A Bone marrow biopsy B Hemoglobin electrophoresis C Serum ferritin D Serum haptoglobin E Serum iron F Serum transferrin

C Serum ferritin With RBC microcytosis, iron deficiency anemia must be considered. It could be a nutritional deficiency in children and pregnant women, but more likely is due to chronic blood loss in adults. The ferritin concentration is a measure of storage iron because it is derived from the total body storage pool in the liver, spleen, and marrow. About 80% of functional body iron is contained in hemoglobin; the remainder is in muscle myoglobin. Individuals with severe liver disease can have an elevated serum ferritin level because of its release from liver stores. A bone marrow biopsy specimen provides a good indication of iron stores because the iron stain of the marrow shows hemosiderin in macrophages, but such a biopsy is an expensive procedure. Some patients with hemoglobinopathies, such as β-thalassemias, also can have a microcytic anemia, but this is far less common than iron deficiency. The serum haptoglobin level is decreased with intravascular hemolysis, but the anemia is normocytic because the iron can be recycled. The serum iron concentration or transferrin level by itself gives no indication of iron stores because in anemia of chronic disease, the patient's iron level can be normal to low, and the transferrin levels also can be normal to low, but iron stores are increased. Transferrin, a serum transport protein for iron, usually has about 33% iron saturation

Since childhood, a 30-year-old man has been easily fatigued with minimal exercise. Laboratory studies show hypochromic microcytic anemia. Hemoglobin electrophoresis reveals decreased Hgb A1 with increased Hgb A2 and Hgb F. His serum ferritin is markedly increased. Which of the following mutations is most likely to be present in the β-globin gene of this man? A New stop codon B Single base insertion, with frameshift C Splice site D Three-base deletion E Trinucleotide repeat

C Splice site This is one mechanism for β+ thalassemia. Because the introns are usually involved, the flanking exons remain, and some normal splicing can occur, so that some β-globin chain synthesis can occur, but not sufficient for adequate hemoglobin production. The other listed mutations lead to a block in translation, with no functional β-globin chain synthesis, typical for βo thalassemia.

A 40-year-old woman has had a week long course of fever and mental confusion. Physical examination shows T 38.2°C, P 100/minute, RR 22/minute, and BP 100/60 mm Hg. She has widespread petechiae of skin and mucosal surfaces. Laboratory studies show her serum urea nitrogen is 52 mg/dL with creatinine 5.3 mg/dL. She has a hemoglobin of 12.2 g/dL, hematocrit 36.8%, MCV 93 fL, platelet count 19,000/microliter, and WBC count 8180/microliter. Schistocytes are seen on her peripheral blood smear. Her prothrombin time, partial thromboplastin time, and D-dimer are not elevated. Which of the following is the most likely diagnosis? A Disseminated intravascular coagulopathy B Idiopathic thrombocytopenic purpura C Thrombotic thrombocytopenic purpura D Trousseau syndrome E Warm autoimmune hemolytic anemia

C Thrombotic thrombocytopenic purpura The pentad of fever, mental changes, renal failure, thrombocytopenia, and microangiopathic hemolytic anemia is characteristic of TTP. Platelets are activated directly, and not the coagulation system as a whole, so that the prothrombin time, partial thromboplastin time, and D-dimer are either not elevated or minimally elevated. The platelet activation leads to formation of hyaline thrombi in small arteries that promotes tissue ischemia in organs such as brain, with consequent neurologic impairment. Platelet transfusion is contraindicated. A - DIC may produce a microangiopathic hemolytic anemia, but there is widespread activation of coagulation so that microthrombi are formed, with markedly elevated prothrombin time, partial thromboplastin time, and D-dimer. DIC typically follows as a complication of a serious disease process, not de novo. B - ITP can lead to thrombocytopenia, but an antibody to platelets is involved, and not platelet activation. D - The hypercoagulable state that is seen with Trousseau syndrome (a paraneoplastic process) results mainly in large venous thrombi, without a microangiopathic hemolytic anemia. E - Autoimmune hemolytic anemias result in hemolysis, but not thrombocytopenia.

A 25-year-old woman has a 3-year history of arthralgias. Physical examination shows no joint deformity, but she ap- pears pale. Laboratory studies show total RBC count of 4.7 million/mm3, hemoglobin of 12.5 g/dL, hematocrit of 37.1%, platelet count of 217,000/mm3, and WBC count of 5890/mm3. The peripheral blood smear shows hypochromic and micro- cytic RBCs. Total serum iron and ferritin levels are normal. Hemoglobin electrophoresis shows 93% hemoglobin A1 with elevated hemoglobin A2 level of 5.8% and hemoglobin F level of 1.2%. What is the most likely diagnosis? A Anemia of chronic disease B Autoimmune hemolytic anemia C β-Thalassemia minor D Infection with Plasmodium vivax E Iron deficiency anemia

C β-Thalassemia minor Although β-thalassemia minor and iron deficiency anemia are both characterized by hypochromic and microcytic RBCs, there is no increase in hemoglobin A2 in iron deficiency states. A normal serum ferritin level also excludes iron deficiency. In contrast to β-thalassemia major, there is usually a mild anemia without major organ dysfunction with β-thalassemia minor. Diseases that produce hemolysis and increase erythropoiesis (e.g., autoimmune hemolytic anemia, malaria) do not alter the composition of β-globin chain production. Anemia of chronic disease may mimic iron deficiency and thalassemia minor with respect to hypochromia and microcytosis; however, anemia of chronic disease is associated with an increase in the serum concentration of ferritin.

A 43-year-old woman of Scandinavian descent complains of constant tiredness, light-headedness, and occasional palpitations and shortness of breath while ascending the stairs. Physical examination shows pallor of the oral mucosa and a raspberry-red tongue (glossitis). Neurologic examination reveals paresthesias, numbness, decreased vibration sensation, and loss of deep tendon reflexes. The results of laboratory studies include hemoglobin of 7.2 g/dL, WBC of 4,500/mL, platelets of 140,000/mL, erythrocyte folate of 220 ng/mL, serum vitamin B12 of 40 pg/mL (normal >200 pg/mL), serum anti-intrinsic factor of 1:128, and serum anti-parietal cell antibody of 1:64. Examination of peripheral blood shows macrocytic anemia, with poikilocytosis of RBCs and hyperseg- mented neutrophils. Atrophic gastritis is diagnosed by gastric biopsy. Bone marrow examination in this patient will reveal which of the following pathologic findings? (A) Absent stainable bone marrow iron (B) Atypical megakaryocytes with fibrosis (C) Hypercellularity with megaloblastic erythroid maturation (D) Hypocellularity with absence of erythroid precursors (E) Myeloid hyperplasia with increased basophils

C. Hypercellularity with megaloblastic erythroid maturation. Pernicious anemia is an autoimmune disorder in which patients develop antibodies directed against gastric parietal cells and intrinsic factor. Parietal cell antibodies lead to atrophic gastritis with achlorhydria. Deficiency of vitamin B12 or folic acid results in megaloblastic anemia. The peripheral blood smear shows macrocytosis and hyper-segmentation of neutrophils. Megaloblastic maturation, characterized by cellular enlargement with asynchronous maturation between the nucleus and cytoplasm, is noted in bone marrow precursors from all lineages. Although the bone marrow tends to be hypercellular, the blood demonstrates pancytopenia because of ineffective hematopoiesis. Neurologic symptoms develop in vitamin B12 deficiency, secondary to degeneration of the posterior and lateral columns of the spinal cord. The other choices are not seen in pernicious anemia. Diagnosis: Megaloblastic anemia, pernicious anemia

Which of the following best describes the pathogenesis of splenomegaly seen in the patient described in Question 12? (A) Amyloidosis (B) Chronic malaria (C) Extramedullary hematopoiesis (D) Infectious mononucleosis (E) Splenic vein thrombosis

C: Extramedullary hematopoiesis. Increased oxygen affinity of hemoglobin F and the underlying anemia impair oxygen delivery and lead to marked bone marrow erythroid hyperplasia. The marrow space is expanded, causing facial and cranial bone deformities. Extramedullary hematopoiesis contributes to splenomegaly and the formation of soft tissue masses. Excess erythropoiesis leads to increased iron absorption, which, together with repeated transfusions, creates iron overload. Excess iron deposition in tissues is a major cause of morbidity and mortality in thalassemic patients. The other choices may cause splenomegaly, but they are not related to β-thalassemia. Diagnosis: Homozygous b-thalassemia, splenomegaly

A 22-year-old woman from a large Italian family is screened for a familial blood disorder. The results of laboratory studies include a hemoglobin of 9.5 g/dL and a smear displaying mild microcytosis, hypochromia, and a few target cells. Hemoglo- bin electrophoresis shows a mild increase in hemoglobin A2 (7.5%). What is the appropriate diagnosis? (A) Anemia of chronic disease (B) G6PD deficiency (C) Heterozygous beta-thalassemia (D) Homozygous beta-thalassemia (E) Silent carrier alpha-thalassemia

C: Heterozygous beta-thalassemia. A normal hemoglobin molecule contains four globin chains, consisting of two α- and two non-α-chains. Three normal variants of hemoglobin are encountered, based on the nature of the non-α-chains. Hemoglobin A (α2 β2) accounts for 95% to 8% of the total hemoglobin in adults; only minor amounts of hemoglobin F (α2 γ2) and hemoglobin A2 (α2 δ2) are present. Heterozygous β-thalassemia is associated with microcytosis and hypochromia, and the degree of microcytosis is disproportionate to the severity of the anemia, which is generally mild. Target cells, basophilic stippling, and a mild increase in hemoglobin A2 are present. Most patients are asymptomatic. Choice D (homozygous β-thalassemia) is a more serious disease and choice E (silent carrier for α-thalassemia) is asymptomatic. Diagnosis: Heterozygous b-thalassemia

A 36-year-old man from China presents with increasing fatigue. He has a 3-year history of tuberculosis, and CBC shows a mild microcytic anemia. Blood work-up demonstrates low serum iron, low iron-binding capacity, and increased serum ferritin. The pathogenesis of anemia in this patient is most likely caused by which of the following mechanisms? (A) Clonal stem cell defect (B) Hypoxemia (C) Impaired utilization of iron from storage sites (D) Synthesis of structurally abnormal globin chains (E) Toxic damage to bone marrow stem cells

C: Impaired utilization of iron from storage sites. Anemia of chronic disease arises in association with chronic infl ammatory diseases (e.g., tuberculosis and rheumatoid arthritis) and malignant conditions. Chronic disease leads to ineffective use of iron from macrophage stores in the bone marrow, resulting in a functional iron deficiency, although storage iron is normal or even increased. The anemia of chronic disease is mild to moderate, and the red cells are often microcytic. Serum iron levels tend to be reduced. However, in contrast to iron defi ciency anemia, total iron binding capacity also tends to be decreased (as is the serum albumin level). The other choices are not related to anemia of chronic disease. Diagnosis: Anemia of chronic disease

A 78-year-old man presents with increasing fatigue. A CBC shows pancytopenia, with moderate anemia (hemoglobin = 10.5 g/dL) and normochromic, hypochromic RBCs. Mild neu- tropenia and thrombocytopenia are noted. A bone marrow evaluation reveals erythroid hyperplasia with increased iron. A Prussian blue-stained bone marrow aspirate is shown in the image. Which of the following is the appropriate diagnosis? (A) Hairy cell leukemia (B) Multiple myeloma (C) Myelodysplastic syndrome (D) Polycythemia vera (E) Promyelocytic leukemia

C: Myelodysplastic syndrome (MDS). MDS exhibits dysplastic morphologic features in one or more hematopoietic lineages and is accompanied by ineffective hematopoiesis. The disease is most common in the elderly and presents with anemia, neutropenia, and thrombocytopenia. The morphologic classification of MDS is based on the presence of abnormally shaped hematopoietic cells and an increased proportion of myeloblasts. Dysplastic features may be present in one or more hematopoietic lineages. Ringed sideroblasts are common. In this case, a smear of a bone marrow aspirate stained with Prussian blue shows erythroid precursor cells containing iron-laden mitochondria that encircle the nuclei. Ringed sideroblasts are not a feature of the other choices. Diagnosis: Myelodysplastic syndrome

A 24-year-old woman with sickle cell disease is seen in the emergency room for an acute upper respiratory tract infection. Laboratory findings include a severe, normocytic anemia. The patient develops a rapid drop in the hemoglobin level. However, the reticulocyte count is very low (<0.01%). This finding most likely reflects which of the following conditions? (A) Bone marrow failure due to repeated infarction (B) Expected result for the patient's underlying anemia (C) Parvovirus B19 infection (D) Retroperitonealhemorrhage (E) Vitamin B12 deficiency

C: Parvovirus B19 infection. Patients with sickle cell anemia may undergo an aplastic crisis because of infection of the bone marrow by parvovirus B19, which suppresses erythrocyte production. None of the other choices are complications of sickle cell anemia. Diagnosis: Sickle cell anemia, aplastic crisis

A 6-year-old girl is brought into the emergency room after an automobile accident. PE shows bleeding from multiple wounds, and a CBC reveals a normocytic, normochromic anemia. Which of the following indices is most helpful in defining this patient's anemia as normocytic? (A) Hematocrit (B) Hemoglobin (C) Mean corpuscular hemoglobin concentration (D) Mean corpuscular volume (E) Red blood cell count

D. Mean corpuscular volume (MCV). MCV is the index used to measure the volume of a red blood cell. It categorizes RBCs by size. Cells with normal size are called normocytic, smaller cells are termed microcytic, and larger cells are referred to as macrocytic. Mean corpuscular hemoglobin concentration (choice C) measures hemoglobin content. Diagnosis: Anemia

An 83-year-old woman fell and broke her left wrist. She underwent open reduction with internal fixation of the radial head fracture. Postoperatively she received heparin prophylaxis for thromboembolism and diuretics to treat pulmonary edema. Laboratory studies showed: On admission: Hgb 13.2 g/dL, Hct 39.1%, MCV 85 fL, platelet count 209,000/microliter, WBC count 5720/microliter, prothrombin time 13 seconds, partial thromboplastin time 28 seconds 1 week later: Hgb 13 g/dL, Hct 38.8%, MCV 86 fL, platelet count 101,000/microliter, WBC count 6310/microliter, prothrombin time 12 seconds, partial thromboplastin time 27 seconds She suddenly developed difficulty in moving her right arm. What is the most likely cause for her findings? A Hypersplenism B Idiopathic thrombocytopenic purpura C Disseminated intravascular coagulation D Drug-induced thrombocytopenia E Thrombotic-thrombocytopenic purpura

D Drug-induced thrombocytopenia She has heparin-induced thrombocytopenia. In about 5% of patients receiving heparin, antibodies develop to a complex of platelet factor 4 with heparin, and in 5 to 14 days there is a marked drop in platelet count. The feared complication is thrombosis ('white clot' syndrome), which can be arterial or venous, and in this patient a thrombotic stroke is likely to have occurred. Low molecular weight heparins are less likely to have this complication. A - Hypersplenism may account for cytopenias due to sequestration of circulating cells in the big spleen, but this does not account for a sudden large drop in the platelet count. B - Chronic ITP is most common in women under the age of 40. There is an antibody directed at platelets. Bleeding is the most likely manifestation, with petechial hemorrhages noted. It is not likely that this disease would have an abrupt onset. C - DIC results from severe underlying diseases such as sepsis and shock. There is consumption of coagulation factors. Her PT and PTT remained normal. E - TTP often has a pentad of neurologic abnormalities, thrombocytopenia (both of which she has), microangiopathic hemolytic anemia, renal failure, and fever (the last three are not present in this case)

A 65-year-old man diagnosed with follicular non-Hodgkin lymphoma is treated with chemotherapy. He develops fever and cough of a week's duration. On examination, there are bilateral pulmonary rales. A chest radiograph shows diffuse interstitial in- filtrates. A sputum specimen is positive for cytomegalovirus. He develops scleral icterus and Raynaud phenomenon. Laboratory studies show hemoglobin, 10.3 g/dL; hematocrit, 41.3%; MCV, 101 μm3; WBC count, 7600/mm3; and platelet count, 205,000/ mm3. His serum total bilirubin is 6 mg/dL, direct bilirubin is 0.8 mg/dL, and LDH is 1020 U/L. Coombs test is positive. Which of the following is the most likely mechanism for his anemia? A Marrow aplasia caused by chemotherapy B Vitamin K deficiency caused by cytomegalovirus hepatitis C Megaloblastic anemia caused by folate deficiency D Extravascular hemolysis caused by cold agglutinins E Iron deficiency caused by metastases to colon

D Extravascular hemolysis caused by cold agglutinins The findings point to Coombs-positive immune hemolytic anemia with indirect hyperbilirubinemia. Cold agglutinin immunohemolytic anemia can be seen with lymphoid neoplasms and infections such as Mycoplasma, Epstein-Barr virus, HIV, influenza virus, and cytomegalovirus (CMV). IgM binds to RBCs at cooler peripheral body regions and then fixes complement. At warmer central regions, the antibody is eluted, but the complement marks the RBCs for extravascular destruction in the spleen, but there is minimal intravascular hemolysis. The increased RBC turnover increases the MCV and the bilirubin, which is mainly indirect. Chemotherapy can suppress bone marrow production, but more likely all cell lines, and without an immune component. Although this patient has CMV infection, CMV hepatitis would likely increase direct and indirect bilirubin, and not account for anemia. Folate deficiency could account for macrocytosis, but not a positive Coombs test. Non-Hodgkin lymphomas do not often involve colon, but this might account for gastrointestinal bleeding with features of iron deficiency and microcytosis.

A study is conducted to determine what changes in the size of the spleen take place with hematologic disorders. The spleen sizes are estimated from CT scans for adult patients who developed complications of their hematologic disease. For which of the following diseases is the spleen most likely to remain normal in size? A Autoimmune hemolytic anemia B Chronic alcohol abuse C Myeloproliferative disorder D Idiopathic thrombocytopenic purpura E Sickle cell anemia

D Idiopathic thrombocytopenia purpura Though with ITP there are circulating antibodies to platelets that lead to platelet destruction, the spleen itself is usually not enlarged. A - In autoimmune hemolytic anemias, the spleen is often the point of removal of the antibody-coated RBCs, and this increased activity increases the size of the spleen. In some hemoglobinopathies with hemolysis, the spleen is also more active and increased in size. B - The portal hypertension accompanying portal fibrosis or cirrhosis with chronic alcoholism leads to splenomegaly. Alcohol is a marrow toxin that can lead to anemia. C - Myeloproliferative disorders, including CML and myelofibrosis, lead to massive splenomegaly. E - By teenage years, persons with Hgb SS have undergone 'autosplenectomy' from the multiple infarctions, leading to a very small spleen.

A clinical study is performed involving adult patients diagnosed with microangiopathic hemolytic anemia. A sub- group of patients who had fever or diarrhea preceding the initial diagnosis of anemia were excluded. The patients had schistocytes present on peripheral blood smears. Some of these patients were found to have a deficiency of a metalloprotein- ase known as ADAMTS13. Which of the following conditions were the patients with this deficiency most likely to have? A Disseminated intravascular coagulation (DIC) B Hemolytic-uremic syndrome (HUS) C Heparin-induced thrombocytopenia (HIT) D Idiopathic thrombocytopenic purpura (ITP) E Thrombotic thrombocytopenic purpura (TTP)

D Idiopathic thrombocytopenic purpura (ITP) A deficiency of ADAMTS13, from an acquired antibody to this metalloproteinase or a genetic mutation in the encoding gene, can lead to accumulation of large von Willebrand multimers that promote platelet microaggregate formation, resulting in TTP that is marked by a pentad of microangiopathic hemolytic anemia, fever, neurologic changes, thrombocytopenia, and renal failure. DIC results from acquired conditions that promote consumption of coagulation factors, not a metalloproteinase deficiency. HUS is very similar to TTP, but is more likely related to a preceding infectious gastroenteritis with diarrhea. HIT occurs in about 5% of individuals receiving heparin, and the most serious complication is widespread arterial and venous thrombosis. ITP is mainly complicated by bleeding from thrombocytopenia.

A 10-year-old child has experienced multiple episodes of pneumonia and meningitis with septicemia since infancy. Causative organisms include Streptococcus pneumoniae and Haemophilus influenzae. On physical examination, the child has no organomegaly and no deformities. Laboratory studies show hemoglobin of 9.2 g/dL, hematocrit of 27.8%, platelet count of 372,000/mm3, and WBC count of 10,300/mm3. A he- moglobin electrophoresis shows 1% hemoglobin A2, 7% he- moglobin F, and 92% hemoglobin S. Which of the following is the most likely cause of the repeated infections in this child? A Absent endothelial cell expression of adhesion molecules B Diminished hepatic synthesis of complement proteins C Impaired neutrophil production D Loss of normal splenic function E Reduced synthesis of immunoglobulins

D Loss of normal splenic function In sickle cell anemia, the cumulative ischemic damage to the spleen results in autosplenectomy, leaving behind a small fibrotic remnant of this organ. The impaired splenic function and resultant inability to clear bacteria from the bloodstream can occur early in childhood, leading to risk for infection with encapsulated bacterial organisms. Immunodeficiency results from lack of splenic function, not from lack of immunoglobulins. Endothelium can be damaged with sickling, and adhesion between endothelial cells and RBCs is increased in sickle cell anemia. Complement proteins are part of innate immune responses in acute inflammation. There is no impairment in production or function of neutrophils.

A 16-year-old boy notes passage of dark urine. He has a history of multiple bacterial infections and venous thromboses for the past 10 years, including portal vein thrombosis in the previous year. On physical examination, his right leg is swol- len and tender. CBC shows hemoglobin, 9.8 g/dL; hematocrit, 29.9%; MCV, 92 μm3; platelet count, 150,000/mm3; and WBC count, 3800/mm3 with 24% segmented neutrophils, 1% bands, 64% lymphocytes, 10% monocytes, and 1% eosinophils. He has a reticulocytosis, and his serum haptoglobin level is very low. A mutation affecting which of the following gene prod- ucts is most likely to give rise to this clinical condition? A β-Globin chain B Factor V C Glucose-6-phosphate dehydrogenase D Phosphatidylinositol glycan A (PIGA) E Prothrombin G20210A F Spectrin

D Phosphatidylinositol glycan A (PIGA) Paroxysmal nocturnal hemoglobinuria (PNH) is a disorder that results from an acquired stem cell membrane defect produced by a PIGA gene mutation that prevents the membrane expression of certain proteins that require a glycolipid anchor. These include proteins that protect cells from lysis by spontaneously activated complement. As a result, RBCs, granulocytes, and platelets are exquisitely sensitive to the lytic activity of complement. The RBC lysis is intravascular, so patients can have hemoglobinuria (dark urine). Defects in platelet function are believed to be responsible for venous thrombosis. Recurrent infections can be caused by impaired leukocyte functions. Patients with PNH may develop acute leukemia or aplastic anemia as complications. Mutations in the β-globin chain can give rise to hemoglobinopathies such as sickle cell anemia. Patients with factor V (Leiden) and prothrombin G20210A mutations can present with thromboses, but there is no anemia or leukopenia. Patients with glucose-6-phosphate dehydrogenase (G6PD) deficiency have an episodic course from exposure to agents such as drugs that induce hemolysis. Spectrin mutations give rise to hereditary spherocytosis.

In an epidemiologic study of anemias, the findings show that there is an increased prevalence of anemia in individu- als of West African ancestry. By hemoglobin electrophoresis, some individuals within this region have increased hemoglo- bin S levels. The same regions also have a high prevalence of an infectious disease. Which of the following infectious agents is most likely to be endemic in the region where such anemia shows increased prevalence? A Borrelia burgdorferi B Clostridium perfringens C Cryptococcus neoformans D Plasmodium falciparum E Treponema pallidum

D Plasmodium falciparum Throughout human history, malaria has influenced the increasing gene frequency of hemoglobin S. Individuals who are heterozygous for hemoglobin S have the sickle cell trait. They are more resistant to malaria, particularly the most malignant form caused by P. falciparum, because the parasites grow poorly or die at low oxygen concentrations, perhaps because of low potassium levels caused by potassium efflux from RBCs on hemoglobin sickling. The malarial parasite has difficulty completing its life cycle, even in cells with moderate amounts of hemoglobin S, as found in heterozygotes, giving a selective advantage to such persons living in endemic areas for falciparum malaria. Borrelia burgdorferi is the spirochete that causes Lyme disease. Clostridium perfringens may produce gas gangrene after soft-tissue injuries. Cryptococcus neoformans can cause granulomatous disease in immunocompromised individuals. Treponema pallidum is the infectious agent causing syphilis. Trypanosoma brucei infection causes sleeping sickness.

A 7-year-old child has had worsening headaches and is obtunded for the past 2 days. Physical examination shows temperature of 39.5° C, pulse of 103/min, respirations of 18/ min, and blood pressure of 90/55 mm Hg. There is bilateral papilledema on funduscopic examination. No focal neuro- logic deficits are noted. Palpation of the abdomen reveals hepatosplenomegaly. Laboratory findings show hemoglobin, 9.5 g/dL; hematocrit, 28.8%; MCV, 101 μm3; platelet count, 145,000/mm3; WBC count, 6920/mm3; Na+, 146 mmol/L; K+, 5.5 mmol/L; Cl-, 106 mmol/L; CO2, 26 mmol/L; creatinine, 2.3 mg/dL; urea nitrogen, 22 mg/dL; LDH, 1095 U/L; and amylase, 45 U/L. The peripheral blood smear is shown in the figure. What infectious agent is most likely to produce these findings? A Babesia microti B Borrelia burgdorferi C Leishmania donovani D Plasmodium falciparum E Trypanosoma brucei F Wuchereria bancrofti

D Plasmodium falciparum This child has cerebral malaria, and the smear shows numerous ring forms of the parasites in RBCs. Infection occurs via the bite of an Anopheles mosquito. Malarial parasites infect RBCs, causing hemolysis and anemia. Falciparum malaria is the worst form. The parasites tend to be released from cells at periodic intervals, leading to periodic fever and chills. The parasites adhere to the vascular endothelium and lead to ischemia in various organs, including the brain with consequent acute cerebral edema. There is hemolytic anemia. The liver and spleen become progressively enlarged. Babesiosis is a rare, tick-borne disease found in the northeastern United States, which can produce a hemolytic anemia, but the organisms produce a classic "tetrad" in RBCs. Lyme disease, caused by Borrelia burgdorferi, is best known for producing chronic arthritis, but meningoencephalitis, neuritis, and neuropathy may complicate this illness. Leishmaniasis, caused by Leishmania donovani, produces mainly visceral disease without cerebral findings. Sleeping sickness, caused by Trypanosoma brucei, can be a chronic disease or a more acute disease causing brain dysfunction (sleeping sickness). Wuchereria bancrofti

An 18-year-old woman from Copenhagen, Denmark, has had malaise and a low-grade fever for the past week, along with arthralgias. On physical examination, she appears very pale, except for a bright red malar facial rash. She has a history of chronic anemia, and spherocytes are observed on a peripheral blood smear. Her hematocrit, which normally ranges from 35% to 38%, is now 28%, and the reticulocyte count is very low. The serum bilirubin level is 0.9 mg/dL. Which of the following events is most likely to have occurred in this patient? A Accelerated extravascular hemolysis in the spleen B Development of anti-RBC antibodies C Disseminated intravascular coagulation D Reduced erythropoiesis from parvovirus infection E Superimposed dietary iron deficiency

D Reduced erythropoiesis from parvovirus infection Patients with hereditary spherocytosis may have an aplastic crisis precipitated by a parvovirus infection. In adults who do not have a defect in normal RBC production, such as hereditary spherocytosis or sickle cell anemia, or who are not immunosuppressed, parvovirus infection is self-limited and often goes unnoticed. When there is an underlying RBC production defect, then RBC production is shut down by parvovirus, and there is no reticulocytosis. Disseminated intravascular coagulation gives rise to thrombocytopenia, bleeding, and the appearance of fragmented RBCs in the blood smear. Reticulocytosis would be prominent with hemolysis and with RBC antibodies. Iron deficiency does not occur in hemolytic anemias because the iron that is released from hemolyzed cells is reused.

A healthy 19-year-old woman suffered blunt abdominal trauma in a motor vehicle accident. On admission to the hos- pital, her initial hematocrit was 33%, but over the next hour, it decreased to 28%. A paracentesis yielded serosanguineous fluid. She was taken to surgery, where a liver laceration was repaired, and 1 L of bloody fluid was removed from the peritoneal cavity. She remained stable. A CBC performed 3 days later is most likely to show which of the following morpho- logic findings in the peripheral blood? A Basophilic stippling of red cells B Hypochromic red cells C Leukoerythroblastosis D Reticulocytosis E Schistocytosis

D Reticulocytosis The acute blood loss, in this case intraperitoneal hemorrhage, results in a reticulocytosis from marrow stimulation by anemia. Basophilic stippling of RBCs suggests a marrow injury, such as with a drug or toxin. Hypochromic RBCs occur in iron deficiency and thalassemias, both associated with reduced hemoglobin synthesis. Acute blood loss does not give rise to iron deficiency if iron stores and diet are adequate. Leukoerythroblastosis is typical of a myelophthisic process in the marrow, with both immature WBCs (myelocytes) and RBCs (nucleated forms) present. Schistocytes suggest a microangiopathic hemolytic anemia, which can accompany shock or sepsis.

In a study of idiopathic aplastic anemia, patients are found who have premature senescence of hematopoietic stem cells. Their hematopoietic cells have normal morphology, but there are fewer cells in myeloid, erythroid, and megakaryo- cytic cell lines. Which of the following enzymes is most likely deficient in their marrow stem cells? A Alkaline phosphatase B Metalloproteinase C Pyruvate kinase D Telomerase E Tyrosine kinase

D Telomerase Telomerase is normally present in continuously divi ding cells, such as hematopoietic stem cells, to prevent shortening of chromosomal telomeres; otherwise, shortened chromosomes cannot divide properly. Both genetic and acquired forms of aplastic anemia have been found that exhibit this mechanism. Alkaline phosphatase participates in bone remodeling. A deficiency of ADAMTS13, a metalloproteinase, can lead to accumulation of large von Willebrand multimers that promote platelet microaggregate formation, resulting in thrombotic thrombocytopenic purpura (TTP). Pyruvate kinase deficiency is one rare form of congenital anemia. Tyrosine kinases participate in cell growth regulation, and are better known to be involved with myeloproliferative disorders.

An 18-year-old man moves from sea level to an elevation of 2,400 m to train as a skier. The increased requirement for oxygen delivery to tissues at the higher elevation stimulates the synthesis of a renal hormone (erythropoietin), which targets hematopoietic stem cells in the bone marrow. Erythropoietin promotes the survival of early erythroid progenitor cells primarily through which of the following mechanisms? (A) Altered cell-matrix adhesion (B) Downregulation of p53 (C) Enhanced glucose uptake (D) Inhibition of apoptosis (E) Stimulation of globin biosynthesis

D: Inhibition of apoptosis. Recent studies indicate that erythropoietin promotes the survival of early erythroid progenitor cells through an inhibition of the default apoptosis pathway. Thus, this hormone rescues stem cells that are otherwise fated to undergo programmed cell death. None of the other choices are known to control the expansion of hematopoietic stem cell colonies in the bone marrow. Diagnosis: Erythropoiesis

A 10-year-old black girl is brought to the emergency room. She complains of severe pain in her chest, abdomen, and bones. Physical examination reveals jaundice and anemia. Her par- ents state that she has been anemic since birth. A CBC shows normocytic anemia with marked poikilocytosis. A peripheral blood smear is shown in the image. Hemoglobin electropho- resis demonstrates hemoglobin S. This child's chest and bone pain is most likely caused by which of the following mechanisms? (A) Amyloidosis (B) Coagulopathy (C) Infection (D) Ischemia (E) Vasculitis

D: Ischemia. Sickle cell disease is characterized by the presence of an abnormal hemoglobin (hemoglobin S). Erythrocyte sickling is initially reversible with deoxygenation, but after several cycles of sickling and unsickling, the process becomes irreversible. Irreversibly sickled cells display a rearrangement of phospholipids between the outer and inner monolayers of the cell membrane. The erythrocytes are no longer deformable and are more adherent to endothelial cells, which are properties that predispose to thrombosis of small blood vessels. The resulting vascular occlusions lead to widespread ischemic complications, which are associated with severe pain, especially in the chest, abdomen, and bones. Diagnosis: Sickle cell disease

A 10-year-old girl has exhibited increasing sluggishness with poorer performance in school over the past year. She has not had increased numbers of infections. The child now complains of headaches. A physical examination shows no hepatosplenomegaly or lymphadenopathy. A CBC shows: Hgb 11.8 g/dL, Hct 33.9%, MCV 71 fL, platelet count 293,000/microliter, and WBC count 8160/microliter. Examination of her peripheral blood smear shows basophilic stippling of erythrocytes. The serum haptoglobin is 5 mg/dL. Which of the following laboratory test findings is most likely to be present in this girl A Hemoglobin S on electrophoresis B Increased osmotic fragility C Positive direct Coombs test D Decreased serum iron E Elevated free erythrocyte protoporphyrin

E Elevated free erythrocyte protoporphyrin She has lead poisoning. Lead inhibits incorporation of iron into heme, leading to elevated zinc protoporphyrin and free erythrocyte protoporphyrin. The diminished heme synthesis leads to a hypochromic, microcytic anemia, and there may be mild hemolysis. Basophilic stippling can be an indicator of toxic injury to RBCs. A - Hemoglobin S leads to erythrocyte sickling and to the appearance of nucleated RBCs in the peripheral blood. B - Increased osmotic fragility is a feature of hereditary spherocytosis. C - A positive Coombs test suggests immune-related hemolysis. D - Though this is a hypochromic, microcytic anemia, the basophilic stippling suggests a toxic injury, not iron deficiency.

A 25-year-old African-American man is given anti-malarial prophylaxis for a trip to West Africa. Over the next week he develops increasing fatigue. On physical examination there are no abnormal findings. Laboratory studies show a hematocrit of 30%. Examination of his peripheral blood smear shows red blood cells with numerous Heinz bodies. There is a family history of this disorder, with males, but not females, affected. Which of the following is the most likely diagnosis? A Beta-thalassemia B Sickle cell anemia C Alpha-thalassemia D Hereditary spherocytosis E G6PD deficiency

E G6PD deficiency He has glucose-6-phosphate dehydrogenase (G6PD) deficiency, which can result in a hemolytic anemia on exposure to oxidizing agents including certain drugs such as antimalarials. This is an X-linked disorder. The Heinz bodies within the RBCs are formed from denatured hemoglobin. A - Beta-thalassemias vary in severity, but generally are chronic conditions without hemolysis. B - Sickle cell anemia is marked by sickling crises brought on by decreased oxygenation, not drugs. C - Alpha-thalassemias tend to be mild in Africans because only 1 or 2 genes on separate members of a pair of chromosomes is involved. D - Hereditary spherocytosis is most likely to be seen in northern Europeans, and there can be aplastic crises brought on by parvovirus infection, not drugs..

A 78-year-old woman has developed increasing dyspnea for the past 2 weeks. On physical examination she has diffuse rales in all lung fields. Scleral icterus is noted. A CBC shows Hgb 7.1, Hct 22.2, MCV 93 fL, platelet count 205,000/microliter, and WBC count 6500/microliter with differential count of 60 segs, 4 bands, 25 lymphs, 9 monos, and 2 eos with 10 nucleated RBCs/100 WBCs. Which of the following is the most likely diagnosis? A Iron deficiency anemia B Pernicious anemia C Anemia of chronic disease D Sickle cell anemia E Hemolytic anemia

E Hemolytic anemia She may have an autoimmune hemolytic anemia, and may have an underlying malignancy such as lymphoma. The icterus is consistent with hyperbilirubinemia from hemolysis. Nucleated RBCs are consistent with high marrow output of erythroid cells. The marked anemia has led to high output congestive heart failure with pulmonary edema. A - An iron deficiency anemia would not be accompanied by nucleated RBC's. The MCV indicates that the RBC's are not microcytic. B - The MCV is not increased. C - Anemia of chronic disease is not accompanied by nucleated RBC's. D - It would be unusual for a patient with sickle cell anemia to live this long.

A 53-year-old man has had constant dull pain in his lower right back for the past 4 months. On physical examination there is tenderness on percussion of his right costovertebral angle. An abdominal CT scan reveals a 6 cm mass in the upper pole of the right kidney. A CBC shows: Hgb 21.3 g/dL, Hct 64.0%, MCV 96 fL, platelet count 199,000/microliter, and WBC count 8230/microliter. Serum chemistries include glucose 77 mg/dL, urea nitrogen 17 mg/dL, and creatinine 1.1 mg/dL. Which of the following is the most likely cause for his findings? A Polycythemia rubra vera B Erythroleukemia C Hemophilia A D Diabetes insipidus E Increased erythropoietin

E Increased erythropoietin He has a secondary form of polycythemia. Renal cell carcinomas are known to secrete erythropoietin and lead to this paraneoplastic effect with polycythemia.

A clinical study is performed with subjects who are adults found to have anemia. Their clinical histories and laboratory findings are reviewed. It is observed that ingestion of a drug preceded development of the anemia in some of the subjects, but not in others. Which of the following conditions is most likely to be found in persons without a history of drug ingestion? A G6PD deficiency B Autoimmune hemolytic anemia C Macrocytic anemia D Aplastic anemia E Microcytic anemia

E Microcytic anemia Microcytic anemia is usually the result of an iron deficient state not associated with drug usage. A - Certain oxidant drugs such as quinacrine or primaquine potentiate this disease. B - Certain drugs (such as aldomet) can cause this disease. C - Folate antagonists such as methotrexate can cause this disease. D - Some drugs can cause an aplastic crisis.

A 55-year-old man has had fatigue, fever, and episodes of epistaxis for the past 3 months. On physical examination his temperature is 37.4°C. Laboratory studies show Hgb 12.5 g/dL, Hct 37.6%, MCV 89 fL, platelet count 170,000/microliter, and WBC count 52,000/microliter. Examination of his peripheral blood smear shows large blasts with Auer rods. Which of the following risk factors most likely preceded development of his current illness? A Malaria B Infectious mononucleosis C Diabetes mellitus D Beta-thalassemia E Myelodysplasia

E Myelodysplasia Auer rods are formed of the cytoplasmic granules of the myeloid blasts of acute myelogenous leukemia (AML) and are a typical finding with AML. Myelodysplastic syndromes can precede development of AML, as can some cases of myeloproliferative disorders, paroxysmal nocturnal hemoglobinuria, and chemotherapeutic regimens. A - Malaria is not a risk factor for malignancy. B - Infectious mononucleosis is a self-limited Epstein-Barr virus infection. EBV may be part of some malignant transformations, but not leukemias. C - Diabetes mellitus is not a risk factor for malignancies. D - Hemoglobinopathies do not lead to malignancies.

A 73-year-old man takes no medications and has had no prior major illnesses or surgeries. For the past year, he has be- come increasingly tired and listless. Physical examination shows that he appears pale but has no hepatosplenomegaly and no de- formities. CBC shows hemoglobin, 9.7 g/dL; hematocrit, 29.9%; MCV, 69.7 mm3; RBC count, 4.28 million/mm3; platelet count, 331,000/mm3; and WBC count, 5500/mm3. His peripheral blood smear is shown in the figure. Which of the following is the most likely underlying condition causing this patient's findings? A Autoimmune hemolytic anemia B Chronic alcohol abuse C β-Thalassemia major D Hemophilia A E Occult malignancy F Vitamin B12 deficiency

E Occult malignancy The figure shows RBC hypochromia and microcytosis, consistent with iron deficiency, the most common cause for anemia worldwide, either from nutritional deficiency or chronic blood loss. The lack of iron impairs heme synthesis. The marrow response is to "downsize" the RBCs, resulting in a microcytic and hypochromic anemia. At this patient's age, bleeding from an occult malignancy, such as a colonic adenocarcinoma, should be strongly suspected as the cause of iron deficiency. An autoimmune hemolytic anemia would appear as a normocytic anemia or as a slightly increased MCV with pronounced reticulocytosis. Macrocytosis would accompany a history of chronic alcoholism, probably because of poor diet and folate deficiency. Thalassemias may result in a microcytosis, but β-thalassemia major causes severe anemia soon after birth, and survival to age 73 years is unlikely. By this patient's age, hemophilia A would have resulted in joint problems; because the bleeding is mainly into soft tissues without blood loss, the iron is recycled. Vitamin B12 deficiency also results in a macrocytic anemia.

A 34-year-old woman reports becoming increasingly tired for the past 5 months. On physical examination, she is afebrile and has mild splenomegaly. Laboratory studies show a hemo- globin concentration of 10.7 g/dL and hematocrit of 32.3%. The peripheral blood smear shows spherocytes and rare nucleated RBCs. Direct and indirect Coombs test results are positive at 37° C, although not at 4° C. Which of the following underlying diseases is most likely to be diagnosed in this patient? A Escherichia coli septicemia B Hereditary spherocytosis C Infectious mononucleosis D Mycoplasma pneumoniae infection E Systemic lupus erythematosus

E Systemic lupus erythematosus This patient has a warm autoimmune hemolytic anemia secondary to systemic lupus erythematosus (SLE). A positive Coombs test result indicates the presence of anti- RBC antibodies in the serum and on the RBC surface. Most cases of warm autoimmune hemolytic anemia are idiopathic, but one fourth occur in individuals with an identifiable autoimmune disease, such as SLE; in other cases, drugs are the cause. The immunoglobulin coating the RBCs acts as an opsonin to promote splenic phagocytosis. Nucleated RBCs can be seen in active hemolysis because the marrow compensates by releasing immature RBCs. Septicemia is more likely to lead to a microangiopathic hemolytic anemia. The increased RBC destruction in hereditary spherocytosis is extravascular and not immune mediated. Infections such as mononucleosis and Mycoplasma are associated with cold autoimmune hemolytic anemia (with an elevated cold agglutinin titer).

A 37-year-old woman has experienced abdominal pain and intermittent low-volume diarrhea for the past 3 months. On physical examination, she is afebrile. A stool sample is positive for occult blood. A colonoscopy is performed, and biopsy specimens from the terminal ileum and colon show microscopic findings consistent with Crohn disease. She does not respond to medical therapy, and part of the colon and terminal ileum are removed. She is transfused with 2U of packed RBCs during surgery. Three weeks later, she appears healthy, but complains of easy fatigability. On investigation, CBC findings show hemoglobin of 10.6 g/dL, hematocrit of 31.6%, RBC count of 2.69 million/μL, MCV of 118 μm3, platelet count of 378,000/mm3, and WBC count of 9800/mm3. The reticulocyte count is 0.3%. Which of the following is most likely to produce these hematologic findings? A Anemia of chronic disease B Chronic blood loss C Hemolytic anemia D Myelophthisic anemia E Vitamin B12 deficiency

E Vitamin B12 deficiency The high MCV indicates a marked macrocytosis, greater than expected from reticulocytosis alone. The two best- known causes for such an anemia (also known as megaloblastic anemia when characteristic megaloblastic precursors are seen in the bone marrow) are vitamin B12 and folate deficiency. Because vitamin B12 complexed with intrinsic factor is ab- sorbed in the terminal ileum, its removal can cause vitamin B12 deficiency. Anemia of chronic disease is generally a normo- cytic anemia. Chronic blood loss and iron deficiency produce a microcytic pattern of anemia, as does dietary iron deficiency. Hemolytic anemia is unlikely several weeks after blood transfusion. Inflammatory bowel diseases (e.g., Crohn disease) increase the risk of malignancy, but myelophthisic anemias (from space-occupying lesions of the marrow) are usually nor- mocytic to mildly macrocytic (from reticulocytosis).

Which of the following mechanisms of disease best describes the pathogenesis of anemia in the patient described in Question 4? (A) Bone marrow fibrosis (B) Clonal stem cell abnormality (C) Defective heme synthesis (D) Immune destruction of circulating erythrocytes (E) Impaired DNA synthesis

E: Impaired DNA synthesis. Megaloblastic anemias are caused by impaired DNA synthesis, usually due to a deficiency of either vitamin B12 or folic acid. In the face of defective DNA synthesis, nuclear development is impaired, whereas cytoplasmic maturation proceeds normally. This situation, termed nuclear to cytoplasmic asynchrony, results in the formation of megaloblasts. Because the megaloblastic precursors do not mature enough to be released into the blood, they undergo intramedullary destruction. Pernicious anemia is not related to any of the other choices. Diagnosis: Megaloblastic anemia

Over the next 6 years, the patient described in Question 18 develops multiple splenic infarcts. Which of the following is a common complication of autosplenectomy in this patient? (A) Autoimmunegastritis (B) Cholelithiasis (C) Megaloblasticanemia (D) Membranousnephropathy (E) Pneumonia

E: Pneumonia. The asplenic state associated with sickle cell anemia renders the patient susceptible to infections by encapsulated bacteria, especially Streptococcus pneumoniae. In addtion to splenic infarcts, patients with sickle cell disease frequently develop renal papillary necrosis due to conditions of low oxygen, low pH, and high osmolality in the renal medulla. None of the other choices represent complications of splenectomy. Diagnosis: Sickle cell disease

A 60-year-old man presents with headaches and pruritis. Physical examination reveals splenomegaly but no lymphaden- opathy. A CBC demonstrates elevated hemoglobin of 19.5 g/dL, WBC of 12,800/μL, and platelets of 550,000/mL. The bone marrow displays hypercellularity of all lineages and depletion of marrow iron stores. Which of the following is the most likely diagnosis? (A) Acute myelogenous leukemia (B) Essential thrombocythemia (C) Idiopathic myelofibrosis (D) Occult infection (E) Polycythemia vera

E: Polycythemia vera (PV). PV is a myeloproliferative disease that arises from a single clonal hematopoietic stem cell and results in uncontrolled production of RBCs. The increase in erythrocytes in PV is autonomous and is not regulated by erythropoietin. PV derives from the malignant transformation of a single, hematopoietic stem cell with primary commitment to the erythroid lineage. Proliferation of the neoplastic clone occurs predominantly in the bone marrow but may involve extramedullary sites including the spleen, lymph nodes, and liver (myeloid metaplasia). The bone marrow is hyper cellular with hyperplasia of all elements. The spleen is moderately enlarged, and its cut surface is uniformly dark red, with expansion of the red pulp and obliteration of the white pulp. Acute myelogenous leukemia (choice A) and essential thrombocythemia (choice B) involve the myeloid and megakaryocytic lines, respectively. Idiopathic myelofi brosis (choice C) features marrow collagen deposition (fibrosis). Diagnosis: Polycythemia vera

A 30-year-old, previously healthy man from Lagos, Nigeria, passes dark brown urine 2 days after starting the prophylactic antimalarial drug primaquine. On physical ex- amination, he appears pale and is afebrile. There is no organo- megaly. Laboratory studies show that his serum haptoglobin level is decreased. Which of the following is the most likely explanation of these findings? A Antibody-mediated hemolysis B Impaired DNA synthesis C Impaired globin chain synthesis D Increased susceptibility to complement-induced lysis E Mechanical fragmentation of RBCs as a result of vascular narrowing F Oxidative injury to hemoglobin G Reduced deformability of RBC membrane

F Oxidative injury to hemoglobin Glucose-6-phosphate dehydrogenase (G6PD) deficiency predisposes the hemoglobin in RBCs to oxidative injury from drugs such as primaquine, and can induce hemolysis. Oxidant injury to hemoglobin produces inclusion of denatured hemoglobin within RBCs. The inclusions damage the cell membrane directly, giving rise to intravascular hemolysis. These damaged RBCs have reduced membrane deformability, and they are removed from the circulation by the spleen. The remaining mechanisms listed are not directly drug dependent. Hemolytic anemias with antibody coating RBCs can occur with autoimmune diseases, prior transfusion, and erythroblastosis fetalis. Impaired RBC nuclear maturation occurs as a result of vitamin B12 or folate deficiency. Impaired globin synthesis occurs in thalassemias. Complement lysis is enhanced in paroxysmal nocturnal hemoglobinuria, which results from mutations in the PIGA gene. Mechanical fragmentation of RBCs is typical of microangiopathic hemolytic anemias, such as disseminated intravascular coagulation. Reduced RBC membrane deformability is seen in patients with abnormalities in cytoskeletal proteins, such as spectrin; the latter causes hereditary spherocytosis.

A 28-year-old woman has had a constant feeling of leth- argy since childhood. On physical examination, she is afebrile and has a pulse of 80/min, respirations of 15/min, and blood pressure of 110/70 mm Hg. The spleen tip is palpable, but there is no abdominal pain or tenderness. Laboratory stud- ies show hemoglobin of 11.7 g/dL, platelet count of 159,000/ mm3, and WBC count of 5390/mm3. The peripheral blood smear shows small round erythrocytes that lack a zone of cen- tral pallor. An inherited abnormality in which of the following RBC components best accounts for these findings? A α-Globin chain B β-Globin chain C Carbonic anhydrase D Glucose-6-phosphate dehydrogenase E Heme with porphyrin ring F Spectrin cytoskeletal protein

F Spectrin cytoskeletal protein Hereditary spherocytosis is a condition in which a mutation affects one of several membrane cytoskeletal proteins. Spectrin and related proteins are cytoskeletal proteins that are important in maintaining the RBC shape. These proteins include ankyrin (most common) and band 4.2, which binds spectrin to the transmembrane ion transporter; band 3; and protein 4.1, which binds the "tail" of spectrin to another transmembrane protein, glycophorin A. The abnormal RBCs with such mutant proteins are less deformable, lack central pallor on a peripheral blood smear, and they are sequestered and destroyed in the spleen. Thalassemias with abnormal α-globin or β-globin chains are associated with hypochromic microcytic anemias. Iron deficiency affects the heme portion of hemoglobin, leading to hypochromia and to microcytosis. Carbonic anhydrase in RBCs helps to maintain buffering capacity. Glucose-6-phosphate dehydrogenase (G6PD) deficiency is an X-linked condition that most commonly affects black males. Porphyrias may affect the production of porphyrin rings and may lead to hemolytic anemia along with abdominal pain, neurologic problems, or skin findings.

A 30-year-old man has had pain and burning on urination for the past week. On physical examination, he is febrile and has a pulse of 92/min, respirations of 18/min, and blood pressure of 80/45 mm Hg. Digital rectal examination indicates that he has an enlarged, tender prostate. There is costovertebral angle tenderness on the right. Scattered ecchymoses are present over the trunk and extremities. Laboratory studies show a blood culture positive for Klebsiella pneumoniae. The appearance of the RBCs in a peripheral blood smear is shown in the figure. Which of the following hematologic disorders is he most likely to have? A Autoimmune hemolytic anemia B Hereditary spherocytosis C Iron deficiency anemia D Megaloblastic anemia E Microangiopathic hemolytic anemia

Gram-negative sepsis with widespread endothelial damage causes disseminated intravascular coagulation. The figure shows fragmented RBCs, including "helmet cells," typical of conditions that can produce a microangiopathic hemolytic anemia, such as disseminated intravascular coagulation, thrombocytopenia purpura, systemic lupus erythematous, hemolytic-uremic syndrome, and malignant hypertension. Such fragmented RBCs are called schistocytes. In autoimmune hemolytic anemia, the hemolysis also is extravascular, and spherocytes are sometimes formed. Spherocytes may be present in hereditary spherocytosis, but the RBC destruction is extravascular, and fragmented RBCs do not appear in the peripheral blood. Marked anisocytosis and poikilocytosis can occur in iron deficiency and in megaloblastic anemias, but fragmentation of RBCs is not seen.

Soon after crossing the finish line in a 10-km race, a 31-year-old man collapses. On physical examination, his tem- perature is 40.1° C, pulse is 101/min, respirations are 22/min, and blood pressure is 85/50 mm Hg. He is not perspiring, and his skin shows decreased turgor. Laboratory studies show Na+, 155 mmol/L; K+, 4.6 mmol/L; Cl-, 106 mmol/L; CO2, 27 mmol/L; glucose, 68 mg/dL; creatinine, 1.8 mg/dL; hemoglo- bin, 20.1 g/dL; hematocrit, 60.3%; platelet count, 230,400/mm3; and WBC count, 6830/mm3. What is the most likely diagnosis? A Erythroleukemia B Chronic obstructive pulmonary disease C Diabetes insipidus D Hemoconcentration E Paraneoplastic syndrome F Polycythemia vera

Heat stroke caused by hyperthermia and loss of perspiration from dehydration is producing hemoconcentration with a relative polycythemia (note the elevated serum sodium level). Erythroleukemia is quite rare, and patients with this disorder are too ill to run a race. Chronic obstructive pulmonary disease is a cause of secondary polycythemia from chronic hypoxemia, but it does not produce hemoconcentration. Diabetes insipidus can result from a lack of antidiuretic hormone release, which leads to free water loss and dehydration, but not to hyperthermia. Increased erythropoietin levels are seen in secondary polycythemia, including those associated with chronic hypoxemia (high altitude or lung disease) and those associated with neoplasms secreting erythropoietin (paraneoplastic syndrome). Polycythemia vera is a form of myeloproliferative disorder, in which erythropoietin levels are low, and there is no dehydration.

A 29-year-old woman has had fatigue with dizziness for the past 5 months. On physical examination, she has an ery- thematous malar rash. She has no lymphadenopathy, but there is a palpable spleen tip. She is afebrile. Laboratory studies show hemoglobin, 8.9 g/dL; hematocrit, 27.8%; MCV, 103 μm3; RBC distribution width index, 22; WBC count, 8650/mm3; platelet count, 222,000/mm3; and reticulocyte count, 3.3%. The periph- eral blood smear shows polychromasia, but no schistocytes. Her serum total bilirubin is 3.2 mg/dL with direct bilirubin 0.8 mg/dL, and haptoglobin is 5 mg/dL. Antinuclear antibody and anti-double-stranded DNA tests are positive. What addi- tional laboratory test finding is she most likely to have? A D-dimer 10 μg/mL B Increased RBC osmotic fragility C Positive Coombs test D Serum cobalamin (vitamin B12) 50 pg/mL E Serum ferritin 240 ng/mL

She has a circulating antibody against her RBCs leading to hemolytic anemia. The indirect antiglobulin (Coombs) test detects antibody in the plasma. The direct antiglobulin (Coombs) test detects antibody bound to RBCs. Autoimmune hemolytic anemias can be a feature of autoimmune diseases, such as systemic lupus erythematosus in this woman. Most of the hemolysis is extravascular in the spleen, but some can be intravascular. The reticulocyte count is typically increased (polychromasia) with hemolysis, and serum haptoglobin is diminished. An elevated D-dimer suggests a microangiopathic hemolytic anemia, but she has no schistocytes on the peripheral blood smear. Increased osmotic fragility is a feature of RBCs in paroxysmal nocturnal hemoglobinuria. Her mild macrocytosis indicates increased reticulocytosis, not vitamin B12 deficiency, and hemolysis is not part of pernicious anemia. Increased serum ferritin is typical for anemia of chronic disease (mild increase) or hemochromatosis (marked increase).

A 22-year-old woman after returning from a trip to Africa has experienced febrile episodes over the past 2 weeks. On physical examination, her temperature is 37.5° C, pulse is 82/min, respirations are 18/min, and blood pressure is 105/65 mm Hg. Laboratory studies show hemoglobin of 10.8 g/dL, hematocrit of 32.5%, platelet count of 245,700/mm3, and WBC count of 8320/mm3. The serum haptoglobin level is decreased, and direct and indirect Coombs test results are negative. The reticulocyte count is increased. The prothrombin time is 12 seconds, and the partial thromboplastin time is 31 seconds. She is observed over the next week and found to have tem- perature spikes to 39.1° C, with shaking chills every 48 hours. Infection with which of the following organisms is most likely to cause this patient's illness? A Aspergillus niger B Babesia microti C Dirofilaria immitis D Escherichia coli E Plasmodium vivax F Wuchereria bancrofti

This is benign tertian malaria. The bite of the Anopheles mosquito introduces sporozoites, which travel to the liver to reproduce. The resulting merozoites are released into the bloodstream and infect RBCs. Asexual reproduction within the RBCs yields trophozoites, and periodic hemolysis with release of the parasites produces the characteristic clinical findings. Aspergillus organisms invade blood vessels and cause thrombosis, but hemolysis of RBCs is inconsequential. Babesiosis is far less common than malaria, is endemic to the northeastern United States, and does not produce episodic fevers. Dirofilaria is the heartworm found in dogs, which rarely infects humans and does not cause hemolysis. Similar to other gram-negative bacteria, Escherichia coli can release lipopolysaccharide, which causes severe sepsis and possible disseminated intravascular coagulation, a microangiopathic hemolytic anemia. Wuchereria bancrofti is a nematode that prefers to live in lymphatics.


Conjuntos de estudio relacionados

AP Euro Chapter 16: Enlightenment

View Set

True or false on driving regulations

View Set

POS Chapter 7—Human Rights and Human Security

View Set

CISSP | Test Questions| Domain 1 | Access Control

View Set